Exam 4 Health Assessment

Lakukan tugas rumah & ujian kamu dengan baik sekarang menggunakan Quizwiz!

In which of the following ethnic groups has the lowest incidence of osteoporosis? A) African Americans B) Whites C) Asians D) American Indians

A) African Americans

When using the various instruments to assess an older person's activities of daily living (ADLs), remember that a disadvantage of these instruments includes: A. the reliability of the tools B. self or proxy report of functional activities C. lack of confidentiality during the assessment D. insufficient detail about the deficiencies identified

B. self or proxy report of functional activities

The Lawton IADL instrument is described by which of the following? A. the nurse uses direct observation to implement this tool B. it is designed as a self-report measure of performance rather than ability C. it is not useful in the acute hospital setting D. it is best used for those residing in an institutional setting

B. it is designed as a self-report measure of performance rather than ability

The knee joint is the articulation of three bones, the: A) femur, fibula, and patella. B) femur, radius, and olecranon process. C) fibula, tibia, and patella. D) femur, tibia, and patella.

D) femur, tibia, and patella.

An older person is experiencing an acute change in cognition. You recognize that this disorder is: A. Alzheimer dementia B. attention deficit disorder C. depression D. delirium

D. delirium

Functional assessment

a systematic assessment that includes assessment of an individual's activities of daily living, instrumental activities of daily living, and mobility

During a visit for a school physical, the 13-year-old girl being examined questions the asymmetry of her breasts. The best response is: a) "One breast may grow faster than the other during development." b) "I will give you a referral for a mammogram." c) "You will probably have fibrocystic disease when you are older." d) "This may be an indication of hormonal imbalance. We will check again in 6 months."

a

Examination of the shoulder includes four motions. These are: a) forward flexion, internal rotation, abduction, and external rotation b) abduction, adduction, pronation, and supination c) circumduction, inversion, eversion, and rotation d) elevation, retraction, protraction, and circumduction

a

Gynecomastia is: a) enlargement of the male breast b) presence of "mast" cells in the male breast c) cancer of the male breast d) presence of supernumerary breast on the male chest.

a

The bulge sign is a test for: a) swelling in the suprapatellar pouch b) carpal tunnel syndrome c) Heberden's nodes d) olecranon bursa inflammation

a

Geriatric assessment

multidimensional assessment; physical examination and assessments of mental status, functional status, social and economic status, pain, and physical environment for safety

A 2-year-old child has been brought to the clinic for a health examination. A common finding would be: a. kyphosis b. lordosis c. scoliosis d. no deviation is normal.

b. lordosis

Which of the following women should not be referred to a physician for further evaluation? a) a 26-year-old with multiple nodules palpated in each breast b) a 48-year-old who has a 6-month history of reddened and sore left nipple and areolar area c) a 25-year-old with asymmetric breasts and inversion of nipples since adolescence d) a 64-year-old with ulcerated area at tip of right nipple; no masses, tenderness, or lymph nodes palpated

c

When teaching the breast self-examination, you would inform the woman that the best time to conduct breast self-examination is: a) at the onset of the menstrual period. b) on the 14th day of the menstrual cycle. c) on the 4th to 7th day of the cycle. d) just before the menstrual period.

c

Anterior and posterior stability is provided to the knee joint by the: a. medial and lateral menisci. b. patellar tendon and ligament c. medial collateral ligament and quadriceps muscle. d. anterior and posterior cruciate ligaments

d. anterior and posterior cruciate ligaments

Domains of cognition

domains included in mental status assessments, such as attention, memory, orientation, language, visuospatial skills, and higher cognitive functions

Instrumental activities of daily living

functional abilities necessary for independent community living, such as shopping, meal preparation, housekeeping, laundry, managing finances, taking medications, and using transportation

Depression

lowering a body part

Elevation

raising a body part

Physical performance measures

tests that measure balance, gait, motor coordination, and endurance

Pronation

turning the forearm so that the palm is down

Circumduction

turning the forearm so that the palm is up

Home care

supportive serviced provided in the home; skilled nursing care, primary care, therapy (physical, occupational, speech), social work, nutrition, case management, ADL assistance, durable medical equipment

Activities of daily living

tasks that are necessary for self-care, such as eating/feeding, bathing, grooming, toileting, walking, and transfering

Supination

moving the arm in a circle around the shoulder

Rotation

moving the head around a central axis

Inversion

moving the sole of the foot inward at the ankle

Eversion

moving the sole of the foot outward at the ankle

Statement by the patient would indicate a need for further teaching? A. "The best time to check my breasts is a week before my cycle." B. "I will check my breasts in the shower one week after my cycle." C. "I will exam my breast in a clock-like sequence so that I don't miss any surface area." D. "I will be sure to check my arm pits."

A The week before a woman's cycle is not the best time to check for lumps because the breasts become sore and tender and may also be "lumpy" a week prior to the cycle. Answer 2 is incorrect because the week after a woman's cycle is the best time to do the examination, and the shower allows for ease of palpation. Answer 3 in incorrect because a clock-like sequence is the proper way to perform a BSE. Answer 4 is incorrect because under the arms houses lymph nodes and the axillary tail of Spence, which is a common site for breast cancer.

Bundles of muscle fibers that compose skeletal muscle are identified as: A) fasciculi. B) fasciculations. C) ligaments. D) tendons.

A) fasciculi.

The production of red blood cells in the bone marrow is called: A) hematopoiesis. B) hemolysis. C) hemoptysis. D) hemianopsia.

A) hematopoiesis.

Heberden and Bouchard nodes are hard and nontender and are associated with: A) osteoarthritis. B) rheumatoid arthritis. C) Dupuytren contracture. D) metacarpophalangeal bursitis.

A) osteoarthritis.

The musculoskeletal system functions include: A) protection and storage. B) movement and elimination. C) storage and control. D) propulsion and preservation.

A) protection and storage.

When assessing for the presence of a herniated nucleus pulposus, the examiner would: A) raise each of the patient's legs straight while keeping the knee extended. B) ask the patient to bend over and touch the floor while keeping the legs straight. C) instruct the patient to do a knee bend. D) abduct and adduct the patient's legs while keeping the knee extended.

A) raise each of the patient's legs straight while keeping the knee extended.

Crepitation is an audible sound that is produced by: A) roughened articular surfaces moving over each other. B) tendons or ligaments that slip over bones during motion. C) joints that are stretched when placed in hyperflexion or hyperextension. D) flexion and extension of an inflamed bursa.

A) roughened articular surfaces moving over each other.

An older adult with new-onset delirium usually has: A. a short attention span. B. trouble naming common objects. C. outbursts of violent behavior. D. vertigo.

A. a short attention span. RATIONALE: Delirium manifests as an acute change in cognition that affects the domain of attention. A person with Alzheimer disease may have alterations in word finding and naming objects in addition to memory problems. A patient with posttraumatic stress disorder may exhibit outbursts of violent behavior. Vertigo is rotational spinning caused by neurologic disease in the vestibular apparatus in the ear or in the vestibular nuclei in the brainstem.

Assessment of the social domain includes: A. family relationships B. ability to cook meals C. ability to balance the checkbook and pay bills D. hazards found in the home

A. family relationships

When you perform a functional assessment of an older patient, which is most appropriate? A. observe the patient's ability to perform the tasks B. ask the patient's wife or husband how he or she does when performing tasks C. review the medical record for information on the patient's abilities D. ask the patient's physician for information on the patient's abilities

A. observe the patient's ability to perform the tasks

An older adult's advanced activities of daily living would include: A. recreational activities B. meal preparation C. balancing the checkbook D. self-grooming activities

A. recreational activities

An older person needs to be assessed before going home as to whether he or she is able to go outside alone safely. Which test is best for this assessment? A. up and go test B. performance of activities of daily living test C. older americans resources and services multidimensional functional assessment questionnaire D. Lawton IADL instrument

A. up and go test

A 43-year-old woman is at the clinic for a routine examination. She reports that she has had a breast lump in her right breast for years. Recently, it has begun to change in consistency and is becoming harder. She reports that 5 years ago her physician evaluated the lump and determined that it "was nothing to worry about." The examination validates the presence of a mass in the right upper outer quadrant at 1 o'clock, approximately 5 cm from the nipple. It is firm, mobile, nontender, with borders that are not well defined. The nurse's recommendation to her is: A) "Because of the change in consistency of the lump, it should be further evaluated by a physician." B) "The changes could be related to your menstrual cycles. Keep track of changes in the mass each month." C) "This is probably nothing to worry about because it has been present for years and was determined to be noncancerous at that time." D) "Because you are experiencing no pain and the size has not changed, continue to monitor the lump and return to the clinic in 3 months."

ANS: A A lump that has been present for years and is not exhibiting changes may not be serious but still should be explored. Any recent change or new lump should be evaluated. The other responses are not correct.

During an examination, the nurse notes a supernumerary nipple just under the patient's left breast. The patient tells the nurse that she always thought it was a mole. Which statement about this finding is correct? A) It is a normal variation and not a significant finding. B) It is a significant finding and needs further investigation. C) It also contains glandular tissue and may leak milk during pregnancy and lactation. D) The patient is correct—it is actually a mole that happens to be located under the breast.

ANS: A A supernumerary nipple looks like a mole, but close examination reveals a tiny nipple and areola. It is not a significant finding.

During a speculum inspection of the vagina, the nurse would expect to see what at the end of the vaginal canal? A) Cervix B) Uterus C) Ovaries D) Fallopian tubes

ANS: A At the end of the canal, the uterine cervix projects into the vagina

A 59-year-old patient has been diagnosed with prostatitis and is being seen at the clinic for complaints of burning and pain during urination. He is experiencing: A) dysuria. B) nocturia. C) polyuria. D) hematuria.

ANS: A Dysuria or burning with urination is common with acute cystitis, prostatitis, and urethritis. Nocturia is voiding during the night. Polyuria is voiding in excessive quantities. Hematuria is voiding with blood in the urine

A 14-year-old girl is anxious about not having reached menarche. When taking the history, the nurse should ascertain which of the following? The age: A) she began to develop breasts B) her mother developed breasts C) she began to develop pubic hair D) she began to develop axillary hair.

ANS: A Full development from stage 2 to stage 5 takes an average of 3 years, although the range is 1.5 to 6 years. Pubic hair develops during this time, and axillary hair appears 2 years after the onset of pubic hair. The beginning of breast development precedes menarche by about 2 years. Menarche occurs in breast development stage 3 or 4, usually just after the peak of the adolescent growth spurt, which occurs around age 12 years. See Figure 17-6.

During a health history of a patient who complains of chronic constipation, the patient asks the nurse about high-fiber foods. The nurse relates that an example of a high-fiber food would be: A) broccoli. B) hamburger. C) iceberg lettuce. D) yogurt.

ANS: A High-fiber foods are either soluble type (i.e., beans, prunes, barley, broccoli) and insoluble type (i.e., cereals, wheat germ). The other examples are not considered high-fiber foods.

The mother of a 5-year-old girl tells the nurse that she has noticed her daughter "scratching at her bottom a lot the last few days." During the assessment, the nurse finds redness and raised skin in the anal area. This most likely indicates: A) pinworms. B) chickenpox. C) constipation. D) bacterial infection.

ANS: A In children, pinworms are a common cause of intense itching and irritated anal skin. The other options are not correct.

An older man is concerned about his sexual performance. The nurse knows that in the absence of disease, a withdrawal from sexual activity later in life may be due to: A) side effects of medications. B) decreased libido with aging. C) decreased sperm production. D) decreased pleasure from sexual intercourse.

ANS: A In the absence of disease, a withdrawal from sexual activity may be due to side effects of medications such as antihypertensives, antidepressants, or sedatives. The other options are not correct.

A newborn baby boy is about to have a circumcision. The nurse knows that indications for circumcision include: A) cultural and religious beliefs. B) prevention of testicular cancer. C) improving the sperm count later in life. D) preventing dysuria

ANS: A Indications for circumcision include cultural and religious beliefs, prevention of phimosis and inflammation of the glans penis and foreskin, decreasing the incidence of cancer of the penis, and decreasing the incidence of urinary tract infections in infancy.

When assessing a newborn infant's genitalia, the nurse notices that the genitalia are somewhat engorged. The labia majora are swollen, the clitoris looks large, and the hymen is thick. The vaginal opening is difficult to visualize. The infant's mother states that she is worried about the labia being swollen. The nurse should reply: A) "This is a normal finding in newborns and should resolve within a few weeks." B) "This could indicate an abnormality and may need to be evaluated by a physician." C) "We will need to have estrogen levels evaluated to make sure that they are within normal limits." D) "We will need to keep close watch over the next few days to see if the genitalia decrease in size."

ANS: A It is normal for a newborn's genitalia to be somewhat engorged. A sanguineous vaginal discharge or leukorrhea is normal during the first few weeks because of the maternal estrogen effect. During the early weeks, the genital engorgement resolves, and the labia minora atrophy and remain small until puberty.

During an examination, which tests will the nurse collect to screen for cervical cancer? A) Endocervical specimen, cervical scrape, and vaginal pool B) Endocervical specimen, vaginal pool, and acetic acid wash C) Endocervical specimen, KOH preparation, and acetic acid wash D) Cervical scrape, acetic acid wash, saline mount ("wet prep")

ANS: A Laboratories may vary in method, but usually the test consists of three specimens: endocervical specimen, cervical scrape, and vaginal pool. The other tests (acetic acid wash, KOH preparation, and saline mount) are used to test for sexually transmitted infections

A woman is in the clinic for an annual gynecologic examination. The nurse should plan to begin the interview with the: A) menstrual history because it is generally nonthreatening. B) obstetric history because it is the most important information. C) urinary system history because there may be problems in this area as well. D) sexual history because it will build rapport to discuss this first.

ANS: A Menstrual history is usually nonthreatening; thus it is a good place to start. Obstetric, urinary, and sexual histories are also part of the interview but not necessarily the best topics with which to start.

When performing a genitourinary assessment, the nurse notices that the urethral meatus is positioned ventrally. This finding is: A) called hypospadias. B) the result of phimosis. C) probably due to a stricture. D) often associated with aging.

ANS: A Normally the urethral meatus is positioned just about centrally. Hypospadias is the ventral location of the urethral meatus. The position of the meatus does not change with aging. Phimosis is the inability to retract the foreskin. A stricture is a narrow opening of the meatus.

The nurse is preparing to interview a postmenopausal woman. Which of these statements is true with regard to the history of a postmenopausal woman? A) The nurse should ask a postmenopausal woman if she ever has vaginal bleeding. B) Once a woman reaches menopause, the nurse does not need to ask any further history questions. C) The nurse should screen for monthly breast tenderness. D) Postmenopausal women are not at risk for contracting sexually transmitted infections and thus these questions can be omitted.

ANS: A Postmenopausal bleeding warrants further workup and referral. The other statements are not true.

The nurse is discussing breast self-examination with a postmenopausal woman. The best time for postmenopausal women to perform breast self-examination is: A) the same day every month. B) daily, during the shower or bath. C) 1 week after her menstrual period. D) every year with her annual gynecologic examination.

ANS: A Postmenopausal women are no longer experiencing regular menstrual cycles but need to continue to perform breast self-examination on a monthly basis. Choosing the same day of the month is a helpful reminder to perform breast self-examination.

The nurse is inspecting the scrotum and testes of a 43-year-old man. Which finding would require additional follow-up and evaluation? A) The skin on the scrotum is taut. B) The left testicle hangs lower than the right testicle. C) The scrotal skin has yellowish 1-cm nodules that are firm and nontender. D) The testes move closer to the body in response to cold temperatures.

ANS: A Scrotal swelling may cause the skin to be taut and to display pitting edema. Normal scrotal skin is rugae, and asymmetry is normal with the left scrotal half usually lower than the right. The testes may move closer to the body in response to cold temperatures.

When performing an external genitalia examination of a 10-year-old girl, the nurse notices that there is no pubic hair, and the mons and the labia are covered with fine vellus hair. These findings are consistent with stage _____ of sexual maturity, according to the Sexual Maturity Rating scale. A) 1 B) 2 C) 3 D) 4

ANS: A Sexual Maturity Rating stage 1 is the preadolescent stage. There is no pubic hair. The mons and labia are covered with fine, vellus hair as on the abdomen. See Table 26-1.

A male patient with possible fertility problems asks the nurse where sperm is produced. The nurse knows that sperm production occurs in the: A) testes. B) prostate. C) epididymis. D) vas deferens.

ANS: A Sperm production occurs in the testes, not in the other structures listed.

A 70-year-old man is visiting the clinic for difficulty in passing urine. In the history he indicates he has to urinate frequently, especially at night. He has burning when he urinates and has noticed pain in his back. Given this history, what might the nurse expect to find during the physical assessment? A) Asymmetric, hard, fixed prostate gland B) Occult blood and perianal pain to palpation C) Symmetrically enlarged, soft prostate gland D) A soft nodule protruding from rectal mucosa

ANS: A Subjective symptoms of carcinoma of the prostate include frequency, nocturia, hematuria, weak stream, hesitancy, pain or burning on urination, and continuous pain in lower back, pelvis, and thighs. Objective symptoms of carcinoma of the prostate include a malignant neoplasm often starts as a single hard nodule on the posterior surface, producing asymmetry and a change in consistency. As it invades normal tissue, multiple hard nodules appear, or the entire gland feels stone hard and fixed.

An 11-year-old girl is in the clinic for a sports physical. The nurse notices that she has begun to develop breasts, and during the conversation the girl reveals that she is worried about her development. The nurse should use which of these techniques to best assist the young girl in understanding the expected sequence for development? The nurse should: A) use the Tanner's table on the five stages of sexual development. B) describe her development and compare it with that of other girls her age. C) use Jacobsen's table on expected development on the basis of height and weight data. D) reassure her that her development is within normal limits and should tell her not to worry about the next step.

ANS: A Tanner's table on the five stages of pubic hair development is helpful in teaching girls the expected sequence of sexual development (see Table 26-1). The other responses are not appropriate.

A woman has just learned that she is pregnant. What are some things the nurse should teach her about her breasts? A) She can expect her areolae to become larger and darker in color. B) Breasts may begin secreting milk after the fourth month of pregnancy. C) She should inspect her breasts for visible veins and report this immediately. D) During pregnancy, breast changes are fairly uncommon; most of the changes occur after the birth.

ANS: A The areolae become larger and grow a darker brown as pregnancy progresses, and the tubercles become more prominent. (The brown color fades after lactation, but the areolae never return to the original color). A venous pattern is prominent over the skin surface and does not need to be reported as it is an expected finding. After the fourth month, colostrum, a thick, yellow fluid (precursor to milk) may be expressed from the breasts.

A 62-year-old man is experiencing fever, chills, malaise, urinary frequency, and urgency. He also reports urethral discharge and a dull aching pain in the perineal and rectal area. These symptoms are most consistent with which of the following? A) Prostatitis B) A polyp C) Carcinoma of the prostate D) Benign prostatic hypertrophy (BPH)

ANS: A The common presenting symptoms of prostatitis are fever, chills, malaise, and urinary frequency and urgency. The individual may also have dysuria, urethral discharge, and a dull aching pain in the perineal and rectal area. See Table 25-3 for descriptions of carcinoma of the prostate and BPH. These are not the symptoms of a polyp.

Which of these statements is most appropriate when the nurse is obtaining a genitourinary history from an elderly man? A) "Do you need to get up at night to urinate?" B) "Do you experience nocturnal emissions, or 'wet dreams'?" C) "Do you know how to perform a testicular self-examination?" D) "Has anyone ever touched your genitals when you did not want them to?"

ANS: A The elderly male patient should be asked about the presence of nocturia. This may be due to diuretic medication, fluid retention from mild heart failure or varicose veins, or fluid ingestion 3 hours before bedtime, especially coffee and alcohol. The other questions are more appropriate for younger males.

The nurse is caring for a newborn infant. Thirty hours after birth, the infant passes a dark green meconium stool. The nurse recognizes that this is important because: A) this stool would indicate anal patency. B) the dark green color could indicate occult blood in the stool. C) meconium stool can be reflective of distress in the newborn. D) the newborn should have passed the first stool within 12 hours after birth.

ANS: A The first stool passed by the newborn is dark green meconium and occurs within 24 to 48 hours of birth, indicating anal patency. The other responses are not correct

The nurse is reviewing statistics regarding breast cancer. Which woman, aged 40 years in the United States, has the highest risk for development of breast cancer? A) African-American B) White C) Asian D) American Indian

ANS: A The incidence of breast cancer varies with different cultural groups. White women have a higher incidence of breast cancer than African-American women starting at age 45 years; but African-American women have a higher incidence before age 45 years. Asian, Hispanic, and American Indian women have a lower risk for development of breast cancer (American Cancer Society, 2009-2010).

During an examination the nurse observes a female patient's vestibule and expects to see the: A) urethral meatus and vaginal orifice. B) vaginal orifice and vestibular (Bartholin) glands. C) urethral meatus and paraurethral (Skene) glands. D) paraurethral (Skene) and vestibular (Bartholin) glands.

ANS: A The labial structures encircle a boat-shaped space, or cleft, termed the vestibule. Within it are numerous openings. The urethral meatus and vaginal orifice are visible. The ducts of the paraurethral (Skene) glands and the vestibular (Bartholin) glands are present but not visible.

During an annual physical exam, a 43-year-old patient states that she doesn't perform monthly breast self-examinations (BSE). She tells the nurse that she believes that mammograms "do a much better job than I ever could to find a lump." The nurse should explain to her that: A) BSEs may detect lumps that appear between mammograms. B) breast self-examination is unnecessary until the age of 50 years. C) she is correct, mammography is a good replacement for breast self-examination. D) she doesn't need to perform breast self-examination as long as a physician checks her breasts yearly.

ANS: A The monthly practice of breast self-examination, along with clinical breast examination and mammograms are complementary screening measures. Mammography can reveal cancers too small to be detected by the woman or by the most experienced examiner. However, interval lumps may become palpable between mammograms.

The nurse is assisting with a self-breast examination clinic. Which of these women reflect abnormal findings during the inspection phase of breast examination? A) Woman whose nipples are in different planes (deviated) B) Woman whose left breast is slightly larger than her right C) Nonpregnant woman whose skin is marked with linear striae D) Pregnant woman whose breasts have a fine blue network of veins visible under the skin

ANS: A The nipples should be symmetrically placed on the same plane on the two breasts. With deviation in pointing, an underlying cancer causes fibrosis in the mammary ducts, which pulls the nipple angle toward it. The other examples are normal findings. See Table 17-3.

The nurse is palpating a female patient's breasts during an examination. Which of these positions is most likely to make significant lumps more distinct during breast palpation? A) Supine with arms raised over her head B) Sitting with arms relaxed at the sides C) Supine with arms relaxed at the sides D) Sitting with arms flexed and fingertips touching shoulders

ANS: A The nurse should help the woman to a supine position, tuck a small pad under the side to be palpated, and help the woman raise her arm over her head. These maneuvers will flatten the breast tissue and displace it medially. Any significant lumps will then feel more distinct

The nurse is preparing to palpate the rectum and should use which of these techniques? A) Flex the finger and insert slowly toward the umbilicus. B) Instruct the patient first that this will be a painful procedure. C) Insert an extended index finger at a right angle to the anus. D) Place the finger directly into the anus to overcome the tight sphincter.

ANS: A The nurse should place the pad of the index finger gently against the anal verge. The nurse will feel the sphincter tighten and then relax. As it relaxes, the nurse should flex the tip of the finger and slowly insert it into the anal canal in a direction toward the umbilicus. The nurse should never approach the anus at right angles with the index finger extended—this would cause pain. The nurse should instruct the patient that palpation is not painful but may feel like needing to move the bowels.

A patient is newly diagnosed with benign breast disease. The nurse recognizes that which statement about benign breast disease is true? The presence of benign breast disease: A) makes it harder to examine the breasts. B) frequently turns into cancer in a woman's later years. C) is easily reduced with hormone replacement therapy. D) is usually diagnosed before a woman reaches childbearing age.

ANS: A The presence of benign breast disease (formerly fibrocystic breast disease) makes it harder to examine the breasts; the general lumpiness of the breast conceals a new lump. The other statements are not true.

A new mother calls the clinic to report that part of her left breast is red, swollen, tender, very hot, and hard. She has a fever of 101° F. She has also had symptoms of the flu, such as chills, sweating, and feeling tired. The nurse notices that she has been breastfeeding for 1 month. From her description, what condition does the nurse suspect? A) Mastitis B) Paget's disease C) Plugged milk duct D) Mammary duct ectasia

ANS: A The symptoms describe mastitis, which stems from infection or stasis caused by a plugged duct. A plugged duct does not have infection present. (See Table 17-7.) Refer to Table 17-6 for descriptions of Paget's disease and mammary duct ectasia.

The uterus is usually positioned tilting forward and superior to the bladder. This position is known as: A) anteverted and anteflexed. B) retroverted and anteflexed. C) retroverted and retroflexed. D) superiorverted and anteflexed.

ANS: A The uterus is freely movable, not fixed, and usually tilts forward and superior to the bladder (a position labeled as anteverted and anteflexed).

The nurse is examining a 62-year-old man and notes that he has gynecomastia bilaterally. The nurse should explore his history for which related conditions? Select all that apply. A) Obesity B) Malnutrition C) Hyperthyroidism D) Type 2 diabetes mellitus E) Liver disease F) History of alcohol abuse

ANS: A, C, E, F Gynecomastia occurs with obesity, Cushing's syndrome, liver cirrhosis, adrenal disease, hyperthyroidism, and numerous drugs: alcohol and marijuana use, estrogen treatment for prostate cancer, antibiotics (metronidazole, isoniazid), digoxin, ACE inhibitors, diazepam, and tricyclic antidepressants.

The nurse is reviewing the changes that occur with menopause. Which of these are changes associated with menopause? A) Uterine and ovarian atrophy along with thinning vaginal epithelium B) Ovarian atrophy, increased vaginal secretions, and increasing clitoral size C) Cervical hypertrophy, ovarian atrophy, and increased acidity of vaginal secretions D) Vaginal mucosa fragility, increased acidity of vaginal secretions, and uterine hypertrophy

ANS: A The uterus shrinks because of its decreased myometrium. The ovaries atrophy to 1 to 2 cm and are not palpable after menopause. The sacral ligaments relax, and the pelvic musculature weakens, so the uterus droops. The cervix shrinks and looks paler with a thick glistening epithelium. The vaginal epithelium atrophies, becoming thinner, drier, and itchy. The vaginal pH becomes more alkaline, and secretions are decreased. This results in a fragile mucosal surface that is at risk for bleeding and vaginitis.

During a vaginal examination of a 38-year-old woman, the nurse notices that the vulva and vagina are erythematous and edematous with thick, white, curdlike discharge adhering to the vaginal walls. The woman reports intense pruritus and thick white discharge from her vagina. The nurse knows that these history and physical examination findings are most consistent with which of these conditions? A) Candidiasis B) Trichomoniasis C) Atrophic vaginitis D) Bacterial vaginosis

ANS: A The woman with candidiasis often reports intense pruritus and thick white discharge. The vulva and vagina are erythematous and edematous. The discharge is usually thick, white, and curdlike. Infection with trichomoniasis causes a profuse, watery, gray-green, and frothy discharge. Bacterial vaginosis causes a profuse discharge that has a "foul, fishy, rotten" odor. Atrophic vaginitis may have a mucoid discharge. See Table 26-5 for complete descriptions of each option.

During a physical examination, the nurse finds that a male patient's foreskin is fixed and tight and will not retract over the glans. The nurse recognizes that this condition is: A) phimosis. B) epispadias. C) urethral stricture. D) Peyronie's disease.

ANS: A With phimosis, the foreskin is nonretractable, forming a pointy tip of the penis with a tiny orifice at the end of the glans. The foreskin is advanced and so tight that it is impossible to retract over the glans. This may be congenital or acquired from adhesions related to infection. See Table 24-3 for information on urethral stricture. See Table 24-4 for information on epispadias and Peyronie's disease.

The nurse is performing a digital examination of a patient's prostate gland and notices that characteristics of a normal prostate gland include which of the following? Select all that apply. A) The gland protruding 1 cm into the rectum B) Heart-shaped with a palpable central groove C) Flat with no groove palpable D) Boggy and soft consistency E) Smooth surface, elastic, or rubbery consistency F) Fixed mobility

ANS: A, B, E The size should be 2.5 cm long by 4 cm wide, and it should not protrude more than 1 cm into the rectum. The prostate should be heart-shaped, with a palpable central groove, a smooth surface, and elastic, rubbery consistency. Abnormal findings include a flat shape with no palpable groove, boggy with a soft consistency, and fixed mobility.

A 16-year-old boy is brought to the clinic for a problem that he refused to let his mother see. The nurse examines him, and finds that he has scrotal swelling on the left side. He had the mumps the previous week, and the nurse suspects that he has orchitis. Which of the assessment findings below support this diagnosis? Select all that apply. A) Swollen testis B) Mass does transilluminate C) Mass does not transilluminate D) Nontender upon palpation E) Tender upon palpation F) Scrotal skin is reddened

ANS: A, C, E, F With orchitis, the testis is swollen, with a feeling of weight, and is tender or painful. The mass does not transilluminate, and the scrotal skin is reddened. Transillumination of a mass occurs with a hydrocele, not orchitis.

A 55-year-old man is in the clinic for a yearly check-up. He is worried because his father died of prostate cancer. The nurse knows that which tests should be done at this time? Select all that apply. A) Blood test for prostate-specific antigen B) Urinalysis C) Transrectal ultrasound D) Digital rectal examination E) Prostate biopsy

ANS: A, D Prostate cancer is typically detected by testing the blood for prostate-specific antigen (PSA) or by a digital rectal exam (DRE). It is recommended that both PSA and DRE be offered to men yearly, beginning at age 50 years. If the PSA is elevated, then further lab work or a transrectal ultrasound (TRUS) and biopsy may be recommended.

The nurse is assessing the breasts of a 68-year-old woman and discovers a mass in the upper outer quadrant of the left breast. When assessing this mass, the nurse keeps in mind that characteristics of a cancerous mass include which of the following? Select all that apply. A) Nontender mass B) Dull, heavy pain on palpation C) Rubbery texture and mobile D) Hard, dense, and immobile E) Regular border F) Irregular, poorly delineated border

ANS: A, D, F Cancerous breast masses are solitary, unilateral, nontender, masses. They are solid, hard, dense, and fixed to underlying tissues or skin as cancer becomes invasive. Their borders are irregular and poorly delineated. They are often painless, although the person may have pain. They are most common in upper outer quadrant. A dull, heavy pain on palpation and a mass with a rubbery texture and a regular border are characteristics of benign breast disease.

A woman who is 22 weeks pregnant has a vaginal infection. She tells the nurse that she is afraid that this infection will hurt the fetus. The nurse knows that which of these statements is true? A) If intercourse is avoided, then the risk for infection is minimal. B) A thick mucus plug forms that protects the fetus from infection. C) The acidic pH of vaginal secretions promotes the growth of pathogenic bacteria. D) The mucus plug that forms in the cervical canal is a good medium for bacterial growth.

ANS: B A clot of thick, tenacious mucus forms in the spaces of the cervical canal (the mucus plug), which protects the fetus from infection. Cervical and vaginal secretions increase during pregnancy and are thick, white, and more acidic. The acidic pH keeps pathogenic bacteria from multiplying in the vagina, but the increase in glycogen increases the risk of candidiasis (commonly called a yeast infection) during pregnancy.

During a digital examination of the rectum, the nurse notices that the patient has hard feces in the rectum. The patient complains of feeling "full," has a distended abdomen, and states that she has not had a bowel movement "for several days." The nurse suspects which condition? A) Rectal polyp B) Fecal impaction C) Rectal abscess D) Rectal prolapse

ANS: B A fecal impaction is a collection of hard, desiccated feces in the rectum. The obstruction often results from decreased bowel motility, in which more water is reabsorbed from the stool. See Table 25-2 for descriptions of rectal polyp and abscess; See Table 25-1 for description of rectal prolapse

A 40-year-old black man is in the office for his annual physical. Which statement regarding the prostate-specific antigen (PSA) blood test is true, according to the American Cancer Society? The PSA: A) should be done with this visit. B) should be done at age 45 years. C) should be done at age 50 years. D) is only necessary if there is a family history of prostate cancer.

ANS: B According to the American Cancer Society (2006) the PSA blood test should be done annually for black men beginning at age 45 years, and annually for all other men over age 50 years.

During a discussion for a men's health group, the nurse relates that the group with the highest incidence of prostate cancer is: A) Asian Americans. B) African-Americans. C) American Indians. D) Hispanics.

ANS: B According to the American Cancer Society (2010), African-American men have a higher rate of prostate cancer than other racial groups.

When performing a genital examination on a 25-year-old man, the nurse notices deeply pigmented, wrinkled scrotal skin with large sebaceous follicles. On the basis of this information the nurse would: A) squeeze the glans to check for the presence of discharge. B) consider this a normal finding and proceed with the examination. C) assess the testicles for the presence of masses or painless lumps. D) obtain a more detailed history focusing on any scrotal abnormalities the patient has noticed.

ANS: B After adolescence, the scrotal skin is deeply pigmented and has large sebaceous follicles. The scrotal skin looks corrugated.

During a breast health interview, a patient states that she has noticed pain in her left breast. The nurse's most appropriate response to this would be: A) "Don't worry about the pain; breast cancer is not painful." B) "I would like some more information about the pain in your left breast." C) "Oh, I had pain like that after my son was born; it turned out to be a blocked milk duct." D) "Breast pain is almost always the result of benign breast disease."

ANS: B Breast pain occurs with trauma, inflammation, infection, or benign breast disease. The nurse will need to gather more information about the patient's pain rather than make statements that ignore the patient's concerns.

A 54-year-old woman who has just completed menopause is in the clinic today for a yearly physical examination. Which of these statements should the nurse include in patient education? "A postmenopausal woman: A) is not at any greater risk for heart disease than a younger woman is." B) should be aware that she is at increased risk for dyspareunia because of decreased vaginal secretions." C) has only stopped menstruating; there really are no other significant changes with which she should be concerned." D) is likely to have difficulty with sexual pleasure as a result of drastic changes in the female sexual response cycle."

ANS: B Decreased vaginal secretions leave the vagina dry and at risk for irritation and pain with intercourse (dyspareunia). The other statements are incorrect.

After completing an assessment of a 60-year-old man with a family history of colon cancer, the nurse discusses with him early detection measures for colon cancer. The nurse should mention the need for a(n): A) annual proctoscopy. B) colonoscopy every 10 years. C) fecal test for blood every 6 months. D) digital rectal examinations every 2 years.

ANS: B Early detection measures for colon cancer include a digital rectal examination performed annually after age 50 years, a fecal occult blood test annually after age 50 years, sigmoidoscopy every 5 years or colonoscopy every 10 years after age 50 years; and a PSA blood test annually for men over 50 years old, except black men beginning at age 45 years (American Cancer Society, 2006).

A 55-year-old man is experiencing severe pain of sudden onset in the scrotal area. It is somewhat relieved by elevation. On examination the nurse notices an enlarged, red scrotum that is very tender to palpation. It is difficult to distinguish the epididymis from the testis, and the scrotal skin is thick and edematous. This description is consistent with which of these? A) Varicocele B) Epididymitis C) Spermatocele D) Testicular torsion

ANS: B Epididymitis presents as severe pain of sudden onset in the scrotum that is somewhat relieved by elevation. On examination, the scrotum is enlarged, reddened, and exquisitely tender. The epididymis is enlarged and indurated and may be hard to distinguish from the testis. The overlying scrotal skin may be thick and edematous. See Table 24-6 for more information and for descriptions of the other terms

During an examination, the nurse would expect the cervical os of a woman who has never had children to appear: A) stellate. B) small and round. C) as a horizontal irregular slit. D) everted.

ANS: B The cervical os in a nulliparous woman is small and round. In the parous woman, it is a horizontal, irregular slit that also may show healed lacerations on the sides. See Figure 26-13.

During an interview, a patient reveals that she is pregnant. She states that she is not sure whether she will breastfeed her baby and asks for some information about this. Which of these statements by the nurse is accurate with regard to breastfeeding? A) "Breastfed babies tend to be more colicky." B) "Breastfeeding provides the perfect food and antibodies for your baby." C) "Breastfed babies eat more often than infants on formula." D) "Breastfeeding is second nature and every woman can do it."

ANS: B Exclusively breastfeeding for 6 months provides the perfect food and antibodies for the baby, decreases the risk of ear infections, promotes bonding, and provides relaxation

The nurse is preparing to examine the external genitalia of a school-age girl. Which of these positions would be most appropriate in this situation? A) In the parent's lap B) In a frog-leg position on the examining table C) In the lithotomy position with the feet in stirrups D) Lying flat on the examining table with legs extended

ANS: B For school-age children it is best to place them on the examining table in a frog-leg position. With toddlers and preschoolers, it is best to have the child on the parent's lap in a frog-leg position.

A woman has just been diagnosed with HPV, or genital warts. The nurse should counsel her to receive regular examinations because this virus makes her at a higher risk for _____ cancer. A) uterine B) cervical C) ovarian D) endometrial

ANS: B HPV is the virus responsible for most cases of cervical cancer, not the other options.

A 30-year-old woman is visiting the clinic because of "pain in my bottom when I have a bowel movement." The nurse should assess for which problem? A) Pinworms B) Hemorrhoids C) Colon cancer D) Fecal incontinence

ANS: B Having painful bowel movements, known as dyschezia, may be due to a local condition (hemorrhoid or fissure) or constipation. The other responses are not correct.

The nurse is describing how to perform a testicular self-examination to a patient. Which of these statements is most appropriate? A) "A good time to examine your testicles is just before you take a shower." B) "If you notice an enlarged testicle or a painless lump, call your health care provider." C) "The testicle is egg shaped and movable. It feels firm and has a lumpy consistency." D) "Perform a testicular exam at least once a week to detect the early stages of testicular cancer."

ANS: B If the patient notices a firm painless lump, a hard area, or an overall enlarged testicle, he should call his health care provider for further evaluation. The testicle normally feels rubbery with a smooth surface. A good time to examine the testicles is during the shower or bath, when one's hands are warm and soapy, and the scrotum is warm. It should be performed once a month.

A patient states during the interview that she noticed a new lump in the shower a few days ago. It was on her left breast near her axilla. The nurse should plan to: A) palpate the lump first. B) palpate the unaffected breast first. C) avoid palpating the lump because it could be a cyst, which might rupture. D) palpate the breast with the lump first but plan to palpate the axilla last.

ANS: B If the woman mentions a breast lump she has discovered herself, the nurse should examine the unaffected breast first to learn a baseline of normal consistency for this individual.

The structure that secretes a thin, milky alkaline fluid to enhance the viability of sperm is the: A) Cowper's gland. B) prostate gland. C) median sulcus. D) bulbourethral gland.

ANS: B In men, the prostate gland secretes a thin milky alkaline fluid that enhances sperm viability. The Cowper's glands (also known as bulbourethral glands) secrete a clear, viscid mucus. The median sulcus is a groove dividing the lobes of the prostate gland and does not secrete fluid.

A nurse is assessing a patient's risk of contracting a sexually transmitted infection (STI). An appropriate question to ask would be: A) "You know that it's important to use condoms for protection, right?" B) "Do you use a condom with each episode of sexual intercourse?" C) "Do you have a sexually transmitted infection?" D) "You are aware of the dangers of unprotected sex, aren't you?"

ANS: B In reviewing a patient's risk for sexually transmitted infections, the nurse should ask, in a nonconfrontational manner, whether condoms are used at each episode of sexual intercourse. Asking a person whether he or she has an infection does not address the risk.

A married couple has come to the clinic seeking advice on pregnancy. They have been trying to conceive for 4 months and have not been successful. What should the nurse do first? A) Ascertain whether either of them has been using broad-spectrum antibiotics. B) Explain that couples are considered infertile after 1 year of unprotected intercourse. C) Immediately refer the woman to an expert in pelvic inflammatory disease—the most common cause of infertility. D) Explain that couples are considered infertile after 3 months of engaging in unprotected intercourse and that they will need a referral to a fertility expert.

ANS: B Infertility is considered after 1 year of engaging in unprotected sexual intercourse without conceiving. The other actions are not appropriate

While inspecting a patient's breasts, the nurse finds that the left breast is slightly larger than the right with the presence of Montgomery's glands bilaterally. The nurse should: A) palpate over the Montgomery's glands, checking for drainage. B) consider these normal findings and proceed with the examination. C) ask extensive history questions regarding the woman's breast asymmetry. D) continue with examination and then refer the patient for further evaluation of the Montgomery's glands.

ANS: B Normal findings of the breast include one breast (most often the left) slightly larger than the other and the presence of Montgomery's glands across the areola.

When performing a scrotal assessment, the nurse notices that the scrotal contents transilluminate and show a red glow. On the basis of this finding the nurse would: A) assess the patient for the presence of a hernia. B) suspect the presence of serous fluid in the scrotum. C) consider this normal and proceed with the examination. D) refer the patient for evaluation of a mass in the scrotum.

ANS: B Normal scrotal contents do not transilluminate. Serous fluid does transilluminate and s

The nurse is palpating a female patient's adnexa. The findings include a firm, smooth uterine wall; the ovaries are palpable and feel smooth and firm. The fallopian tube is firm and pulsating. The nurse's most appropriate course of action would be to: A) tell the patient that her examination was normal. B) give her an immediate referral to a gynecologist. C) suggest that she return in a month for a recheck to verify the findings. D) tell the patient that she may have an ovarian cyst that should be evaluated further.

ANS: B Normally the uterine wall feels firm and smooth, with the contour of the fundus rounded. Ovaries are not often palpable, but when they are, they normally feel smooth, firm, and almond shaped and are highly movable, sliding through the fingers. The fallopian tube is not palpable normally. No other mass or pulsation should be felt. Pulsation or palpable fallopian tube suggests ectopic pregnancy, which warrants immediate referral

The mother of a 10-year-old boy asks the nurse to discuss the recognition of puberty. The nurse should reply by saying: A) "Puberty usually begins about age fifteen." B) "The first sign of puberty is enlargement of the testes." C) "Penis size does not increase until about the age of sixteen." D) "The development of pubic hair precedes testicular or penis enlargement."

ANS: B Puberty begins sometime between ages 9 1/2 and 13 1/2 years. The first sign is enlargement of the testes. Next, pubic hair appears and then penis size increases.

During an examination of a 7-year-old girl, the nurse notices that the girl is showing breast budding. What should the nurse do next? A) Ask her if her periods have started. B) Assess the girl's weight and body mass index (BMI). C) Ask the girl's mother at what age she started to develop breasts. D) Nothing; this is a normal finding.

ANS: B Research has shown that girls with overweight or obese BMI levels have a higher occurrence of early onset of breast budding (before age 8 years for African-American girls and age 10 years for white girls) and early menarche.

When assessing the scrotum of a male patient, the nurse notices the presence of multiple firm, nontender, yellow 1-cm nodules. The nurse knows that these nodules are most likely: A) from urethritis. B) sebaceous cysts. C) subcutaneous plaques. D) from inflammation of the epididymis.

ANS: B Sebaceous cysts are commonly found on the scrotum. These are yellowish 1-cm nodules and are firm, nontender, and often multiple. The other options are not correct.

A 50-year-old woman calls the clinic because she has noticed some changes in her body and breasts and wonders if they could be due to the hormone replacement therapy (HRT) she started 3 months ago. The nurse should tell her: A) "Hormone replacement therapy is at such a low dose that side effects are very unusual." B) "Hormone replacement therapy has several side effects, including fluid retention, breast tenderness, and vaginal bleeding." C) "It would be very unusual to have vaginal bleeding with hormone replacement therapy, and I suggest you come in to the clinic immediately to have this evaluated." D) "It sounds as if your dose of estrogen is too high; I think you may need to decrease the amount you are taking and then call back in a week."

ANS: B Side effects of hormone replacement therapy include fluid retention, breast pain, and vaginal bleeding. The other responses are not correct.

When the nurse is discussing sexuality and sexual issues with adolescents, a permission statement helps to convey that it is normal to think or feel a certain way. Which of these is the best example of a permission statement? A) "It is okay that you have become sexually active." B) "Often girls your age have questions about sexual activity. Have you any questions?" C) "If it is okay with you, I'd like to ask you some questions about your sexual history." D) "Often girls your age engage in sexual activity. It is okay to tell me if you have had intercourse."

ANS: B Start with a permission statement, "Often girls your age experience . . . ." This conveys that it is normal to think or feel a certain way, and it is important to relay that the topic is normal and unexceptional.

A 52-year-old patient states that when she sneezes or coughs she "wets herself a little." She is very concerned that something may be wrong with her. The nurse suspects that the problem is: A) dysuria. B) stress incontinence. C) hematuria. D) urge incontinence.

ANS: B Stress incontinence is involuntary urine loss with physical strain, sneezing, or coughing. Dysuria is pain or burning with urination. Hematuria is bleeding with urination. Urge incontinence is involuntary urine loss but it occurs due to an overactive detrusor muscle in the bladder that contracts and causes an urgent need to void

The nurse is performing an examination of the anus and rectum. Which of these statements is correct and important to remember during this examination? A) The rectum is about 8 cm long. B) The anorectal junction cannot be palpated. C) Above the anal canal, the rectum turns anteriorly. D) There are no sensory nerves in the anal canal or rectum.

ANS: B The anal columns are folds of mucosa that extend vertically down from the rectum and end in the anorectal junction. This junction is not palpable, but it is visible on proctoscopy. The rectum is 12 cm long, and just above the anal canal, the rectum dilates and turns posteriorly.

If a patient reports a recent breast infection, then the nurse should expect to find _____ node enlargement. A) nonspecific B) ipsilateral axillary C) contralateral axillary D) inguinal and cervical

ANS: B The breast has extensive lymphatic drainage. Most of the lymph, more than 75%, drains into the ipsilateral, or same side, axillary nodes.

Which of the following statements is true regarding the internal structures of the breast? The breast is: A) mainly muscle, with very little fibrous tissue. B) composed of fibrous, glandular, and adipose tissue. C) composed mostly of milk ducts, known as lactiferous ducts. D) composed of glandular tissue, which supports the breast by attaching to the chest wall.

ANS: B The breast is composed of glandular tissue, fibrous tissue (including the suspensory ligaments), and adipose tissue.

During an internal examination, the nurse notices that the cervix bulges outside the introitus when the patient is asked to strain. The nurse will document this as: A) uterine prolapse, graded first degree. B) uterine prolapse, graded second degree. C) uterine prolapse, graded third degree. D) a normal finding.

ANS: B The cervix should not be found to bulge into the vagina. Uterine prolapse is graded as follows: first degree—cervix appears at introitus with straining; second degree—cervix bulges outside introitus with straining; and third degree—whole uterus protrudes, even without straining (essentially, uterus is inside out).

The external male genital structures include the: A) testis. B) scrotum. C) epididymis. D) vas deferens.

ANS: B The external male genital structures include the penis and scrotum. The testis, epididymis, and vas deferens are internal structures.

Which of these statements about the sphincters is correct? A) The internal sphincter is under voluntary control. B) The external sphincter is under voluntary control. C) Both sphincters remain slightly relaxed at all times. D) The internal sphincter surrounds the external sphincter.

ANS: B The external sphincter surrounds the internal sphincter but also has a small section overriding the tip of the internal sphincter at the opening. The external sphincter is under voluntary control. Except for the passing of feces and gas, the sphincters keep the anal canal tightly closed.

The nurse is palpating the prostate gland through the rectum and notices an abnormal finding if which of these is present? A) Palpable central groove B) Tenderness to palpation C) Heart shape D) Elastic and rubbery consistency

ANS: B The normal prostate gland should feel smooth, elastic, and rubbery; should be slightly movable; should be heart-shaped with a palpable central groove; and should not be tender to palpation.

During the examination portion of a patient's visit, she will be in lithotomy position. Which statement below reflects some things that the nurse can do to make this more comfortable for her? A) Ask her to place her hands and arms behind her head. B) Elevate her head and shoulders to maintain eye contact. C) Allow her to choose to have her feet in the stirrups or have them resting side by side on the edge of the table. D) Allow her to keep her buttocks about 6 inches from the edge of the table to prevent her from feeling as if she will fall off.

ANS: B The nurse should elevate her head and shoulders to maintain eye contact. The patient's arms should be placed at her sides or across the chest, not behind the head, because this position only tightens the abdominal muscles. The feet should be placed into the stirrups, knees apart, and buttocks at the edge of the examining table. Place the stirrups so the legs are not abducted too far

During a breast examination on a female patient, the nurse notices that the nipple is flat, broad, and fixed. The patient states it "started doing that a few months ago." This finding suggests: A) dimpling. B) a retracted nipple. C) nipple inversion. D) deviation in nipple pointing.

ANS: B The retracted nipple looks flatter and broader, like an underlying crater. A recent retraction suggests cancer, which causes fibrosis of the whole duct system and pulls in the nipple. It also may occur with benign lesions such as ectasia of the ducts. The nurse should not confuse retraction with the normal long-standing type of nipple inversion, which has no broadening and is not fixed.

In performing a breast examination, the nurse knows that it is especially important to examine the upper outer quadrant of the breast. The reason for this is that the upper outer quadrant is: A) the largest quadrant of the breast. B) the location of most breast tumors. C) where most of the suspensory ligaments attach. D) more prone to injury and calcifications than other locations in the breast.

ANS: B The upper outer quadrant is the site of most breast tumors. In the upper outer quadrant, the nurse should notice the axillary tail of Spence, the cone-shaped breast tissue that projects up into the axilla, close to the pectoral group of axillary lymph nodes.

The nurse has just completed an inspection of a nulliparous woman's external genitalia. Which of these would be a description of a finding within normal limits? A) Redness of the labia majora B) Multiple nontender sebaceous cysts C) Discharge that is sticky and yellow-green D) Gaping and slightly shriveled labia majora

ANS: B There should be no lesions, except for occasional sebaceous cysts. These are yellowish 1-cm nodules that are firm, nontender, and often multiple. The labia majora are dark pink, moist, and symmetrical; redness indicates inflammation or lesions. Discharge that is sticky and yellow-green may indicate infection. In the nulliparous woman, the labia majora meet in the midline, are symmetric and plump.

During an examination of an aging male, the nurse recognizes that normal changes to expect would be: A) a change in scrotal color. B) a decrease in the size of the penis. C) enlargement of the testes and scrotum. D) an increase in the number of rugae over the scrotal sac.

ANS: B When assessing the genitals of an older man, the nurse may notice thinner, graying pubic hair and a decrease in the size of the penis. The size of the testes may be decreased, they may feel less firm, and the scrotal sac is pendulous with less rugae. There is no change in scrotal color.

A patient calls the clinic for instructions before having a Papanicolaou (Pap) smear. The most appropriate instructions from the nurse are: A) "If you are menstruating, please use pads to avoid placing anything into the vagina." B) "Avoid intercourse, inserting anything into the vagina, or douching within 24 hours of your appointment." C) "If you suspect that you have a vaginal infection, please gather a sample of the discharge to bring with you." D) "We would like you to use a mild saline douche before your examination. You may pick this up in our office."

ANS: B When instructing a patient before a Papanicolaou (Pap) smear is obtained, the nurse should follow these guidelines: Do not obtain during the woman's menses or if a heavy infectious discharge is present. Instruct the woman not to douche, have intercourse, or put anything into the vagina within 24 hours before collecting the specimens. Any specimens will be obtained during the visit, not beforehand.

The nurse is palpating an ovarian mass during an internal examination of a 63-year-old woman. Which findings of the mass's characteristics would suggest the presence of an ovarian cyst? Select all that apply. A) Heavy and solid B) Mobile and fluctuant C) Mobile and solid D) Fixed E) Smooth and round F) Poorly defined

ANS: B, E An ovarian cyst (fluctuant ovarian mass) is usually asymptomatic, and would feel like a smooth, round, fluctuant, mobile, nontender mass on the ovary. A mass that is heavy, solid, fixed, and poorly defined suggests malignancy. A benign mass may feel mobile and solid.

The nurse is preparing for a class in early detection of breast cancer. Which statement is true with regard to breast cancer in African-American women in the United States? A) Breast cancer is not a threat to African-American women. B) African-American women have a lower incidence of regional or distant breast cancer than white women. C) African-American women are more likely to die of breast cancer at any age. D) Breast cancer incidence in African-American women is higher than that of white women after age 45.

ANS: C African-American women have a higher incidence of breast cancer before age 45 years than white women, and are more likely to die of their disease. In addition, African-American women are significantly more likely to be diagnosed with regional or distant breast cancer than are white women. This racial difference in mortality rates may be related to insufficient use of screening measures and lack of access to health care.

During the interview a patient reveals that she has some vaginal discharge. She is worried that it may be a sexually transmitted infection. The nurse's most appropriate response to this would be: A) "Oh, don't worry. Some cyclic vaginal discharge is normal." B) "Have you been engaging in unprotected sexual intercourse?" C) "I'd like some information about the discharge. What color is it?" D) "Have you had any urinary incontinence associated with the discharge?"

ANS: C Ask questions that help the patient reveal more information about her symptoms in a nonthreatening manner. Assess vaginal discharge further by asking about the amount, color, and odor. Normal vaginal discharge is small, clear or cloudy, and always nonirritating.

The changes normally associated with menopause occur generally because the cells in the reproductive tract are: A) aging. B) becoming fibrous. C) estrogen dependent. D) able to respond to estrogen.

ANS: C Because cells in the reproductive tract are estrogen dependent, decreased estrogen levels during menopause bring dramatic physical changes. The other options are not correct.

A patient who is visiting the clinic complains of having "stomach pains for 2 weeks" and describes his stools as being "soft and black" for about the last 10 days. He denies taking any medications. The nurse is aware that these symptoms are most indicative of: A) excessive fat caused by malabsorption. B) increased iron intake resulting from a change in diet. C) occult blood resulting from gastrointestinal bleeding. D) absent bile pigment from liver problems.

ANS: C Black stools may be tarry due to occult blood (melena) from gastrointestinal bleeding or nontarry from ingestion of iron medications (not diet). Excessive fat causes the stool to become frothy; absence of bile pigment causes clay-colored stools.

A woman is in the family planning clinic seeking birth control information. She states that her breasts "change all month long" and that she is worried that this is unusual. What is the nurse's best response? A) Tell her that it is unusual. The breasts of nonpregnant females usually stay pretty much the same all month long. B) Tell her that it is very common for breasts to change in response to stress and that she should assess her life for stressful events. C) Tell her that, because of the changing hormones during the monthly menstrual cycle, cyclic breast changes are common. D) Tell her that breast changes normally occur only during pregnancy and that a pregnancy test is needed at this time.

ANS: C Breasts of the nonpregnant woman change with the ebb and flow of hormones during the monthly menstrual cycle. During the 3 to 4 days before menstruation, the breasts feel full, tight, heavy, and occasionally sore. The breast volume is smallest on days 4 to 7 of the menstrual cycle.

The nurse is performing a genitourinary assessment on a 50-year-old obese male laborer. On examination the nurse notices a painless round swelling close to the pubis in the area of the internal inguinal ring that is easily reduced when the individual is supine. These findings are most consistent with a(n) _____ hernia. A) scrotal B) femoral C) direct inguinal D) indirect inguinal

ANS: C Direct inguinal hernias occur most often in men over the age of 40 years. It is an acquired weakness brought on by heavy lifting, obesity, chronic cough, or ascites. The direct inguinal hernia is usually a painless, round swelling close to the pubis in the area of the internal inguinal ring that is easily reduced when the individual is supine. See Table 24-6 for a description of scrotal hernia. See Table 24-7 for descriptions of femoral hernias and indirect inguinal hernias.

During a genital examination, the nurse notices that a male patient has clusters of small vesicles on the glans, surrounded by erythema. The nurse recognizes that these lesions are: A) Peyronie disease. B) genital warts. C) genital herpes. D) syphilitic cancer.

ANS: C Genital herpes, or HSV-2, infections are indicated with clusters of small vesicles with surrounding erythema, which are often painful and erupt on the glans or foreskin. See Table 24-4 for descriptions of the other options.

An accessory glandular structure for the male genital organs is the: A) testis. B) penis. C) prostate. D) vas deferens.

ANS: C Glandular structures accessory to the male genital organs are the prostate, seminal vesicles, and bulbourethral glands.

A patient has had three pregnancies and two live births. The nurse would record this information as gravida _____, para _____, AB _____. A) 2; 2; 1 B) 3; 2; 0 C) 3; 2; 1 D) 3; 3; 1

ANS: C Gravida is number of pregnancies. Para is number of births. Abortions are interrupted pregnancies, including elective abortions and spontaneous miscarriages.

In examining a 70-year-old male patient, the nurse notices that he has bilateral gynecomastia. Which of the following describes the nurse's best course of action? A) Recommend that he make an appointment with his physician for a mammogram. B) Ignore it; it is not unusual for men to have benign breast enlargement. C) Explain that this condition may be the result of hormonal changes and recommend that he see his physician. D) Tell him that gynecomastia in men is usually associated with prostate enlargement and recommend that he be screened thoroughly.

ANS: C Gynecomastia may reappear in the aging male and may be due to testosterone deficiency.

During an external genitalia examination of a woman, the nurse notices several lesions around the vulva. The lesions are pink, moist, soft, and pointed papules. The patient states that she is not aware of any problems in that area. The nurse recognizes that these lesions may be: A) syphilitic chancre. B) herpes simplex virus type 2 (herpes genitalis). C) human papillomavirus (HPV), or genital warts. D) pediculosis pubis (crab lice).

ANS: C HPV lesions are painless, warty growths that the woman may not notice. Lesions are pink or flesh colored, soft, pointed, moist, warty papules that occur in single or multiple cauliflower-like patches around the vulva, introitus, anus, vagina, or cervix. Herpetic lesions are painful clusters of small, shallow vesicles with surrounding erythema. Syphilitic chancres begin as a solitary silvery papule that erodes to a red, round or oval, superficial ulcer with a yellowish discharge. Pediculosis pubis causes severe perineal itching and excoriations and erythematous areas. See Table 26-2.

During the interview with a female patient, the nurse gathers data that indicate that the patient is perimenopausal. Which of these statements made by this patient leads to this conclusion? A) "I have noticed that my muscles ache at night when I go to bed." B) "I will be very happy when I can stop worrying about having a period." C) "I have been noticing that I sweat a lot more than I used to, especially at night." D) "I have only been pregnant twice, but both times I had breast tenderness as my first symptom."

ANS: C Hormone shifts occur during the perimenopausal period, and associated symptoms of menopause may occur, such as hot flashes, night sweats, numbness and tingling, headache, palpitations, drenching sweats, mood swings, vaginal dryness, and itching. The other responses are not correct.

The nurse has palpated a lump in a female patient's right breast. The nurse documents this as a small, round, firm, distinct, lump located at 2 o'clock, 2 cm from the nipple. It is nontender and fixed. There is no associated retraction of skin or nipple, no erythema, and no axillary lymphadenopathy. Which of these statements reveals the information that is missing from the documentation? It is missing information about: A) the shape of the lump. B) the lump's consistency. C) the size of the lump. D) whether the lump is solitary or multiple.

ANS: C If the nurse feels a lump or mass, he or she should note these characteristics: (1) location, (2) size—judge in centimeters in three dimensions: width × length × thickness, (3) shape, (4) consistency, (5) motility, (6) distinctness, (7) nipple, (8) the skin over the lump, (9) tenderness, and (10) lymphadenopathy.

A 2-month-old uncircumcised infant has been brought to the clinic for a well-baby checkup. How would the nurse proceed with the genital examination? A) Elicit the cremasteric reflex. B) Assess the glans for redness or lesions. C) Avoid retracting the foreskin until the infant is 3 months old. D) Note any dirt or smegma that has collected under the foreskin.

ANS: C If uncircumcised, then the foreskin is normally tight during the first 3 months and should not be retracted because of the risk of tearing the membrane attaching the foreskin to the shaft. The other options are not correct.

During an examination of an aging male, the nurse recognizes that normal changes to expect would be: A) enlarged scrotal sac. B) increased pubic hair. C) decreased penis size. D) increased rugae over the scrotum.

ANS: C In the aging male the amount of pubic hair decreases, the penis size decreases, and there is a decrease in the rugae over the scrotal sac. The scrotal sac does not enlarge.

During a physical examination, a 45-year-old woman states that she has had a crusty, itchy rash on her breast for about 2 weeks. In trying to find the cause of the rash, which of these would be important for the nurse to determine? A) Is the rash raised and red? B) Does it appear to be cyclic? C) Where did it first appear—on the nipple, the areola, or the surrounding skin? D) What was she doing when she first noticed the rash, and do her actions make it worse?

ANS: C It is important for the nurse to determine where the rash first appeared. Paget's disease starts with a small crust on the nipple apex and then spreads to the areola. Eczema or other dermatitis rarely starts at nipple unless it results from breastfeeding. It usually starts on the areola or surrounding skin and then spreads to the nipple. See Table 17-6.

The nurse is aware that which of these statements is true regarding the incidence of testicular cancer? A) Testicular cancer is the most common cancer in men aged 30 to 50 years. B) The early symptoms of testicular cancer are pain and induration. C) Men with a history of cryptorchidism are at greatest risk for development of testicular cancer. D) The cure rate for testicular cancer is low.

ANS: C Men with undescended testicles (cryptorchidism) are at greatest risk for development of testicular cancer. The overall incidence of testicular cancer is rare. Testicular cancer has no early symptoms. When detected early and treated before metastasis, the cure rate is almost 100%.

A 2-year-old boy has been diagnosed with "physiologic cryptorchidism." Given this diagnosis, during assessment the nurse will most likely observe: A) testes that are hard and painful to palpation. B) an atrophic scrotum and absence of the testis bilaterally. C) an absence of the testis in the scrotum, but the testis can be milked down. D) testes that migrate into the abdomen when the child squats or sits cross-legged.

ANS: C Migratory testes (physiologic cryptorchidism) are common because of the strength of the cremasteric reflex and the small mass of the prepubertal testes. The affected side has a normally developed scrotum and the testis can be milked down. The other responses are not correct.

During a bimanual examination, the nurse detects a solid tumor on the ovary that is heavy and fixed, with a poorly defined mass. This finding is suggestive of: A) an ovarian cyst. B) endometriosis. C) ovarian cancer. D) an ectopic pregnancy.

ANS: C Ovarian tumors that are solid, heavy, and fixed, with poorly defined mass are suggestive of malignancy. Benign masses may feel mobile and solid. An ovarian cyst may feel smooth, round, fluctuant, mobile, and nontender. With an ectopic pregnancy, the examiner may feel a palpable, tender pelvic mass that is solid, mobile, and unilateral. Endometriosis may have masses (in various locations in the pelvic area) that are small, firm, nodular and tender to palpation, with enlarged ovaries.

The nurse is examining a 35-year-old female patient. During the history, the nurse notices that she has had two term pregnancies, and both babies were delivered vaginally. During the internal examination the nurse observes that the cervical os is a horizontal slit with some healed lacerations and that the cervix has some nabothian cysts that are small, smooth, and yellow. In addition, the nurse notices that the cervical surface is granular and red, especially around the os. Finally, the nurse notices the presence of stringy, opaque, odorless secretions. Which of these findings are abnormal? A) Nabothian cysts are present. B) The cervical os is a horizontal slit. C) The cervical surface is granular and red. D) Stringy and opaque secretions are present.

ANS: C Normal findings: Nabothian cysts may be present on the cervix after childbirth. The cervical os is a horizontal, irregular slit in the parous woman. Secretions vary according to the day of the menstrual cycle, and may be clear and thin or thick, opaque, and stringy. The surface is normally smooth, but cervical eversion, or ectropion may occur where the endocervical canal is "rolled out." Abnormal finding: The cervical surface should not be reddened or granular, which may indicate a lesion.

When performing the bimanual examination, the nurse notices that the cervix feels smooth and firm, is round, and is fixed in place (does not move). When cervical palpation is performed, the patient complains of some pain. The nurse's interpretation of these results should be which of these? A) These findings are all within normal limits. B) The cervical consistency should be soft and velvety—not firm. C) The cervix should move when palpated; an immobile cervix may indicate malignancy. D) Pain may occur during palpation of the cervix.

ANS: C Normally the cervix feels smooth and firm, as the consistency of the tip of the nose. It softens and feels velvety at 5 to 6 weeks of pregnancy (Goodell's sign). The cervix should be evenly rounded. With a finger on either side, the examiner should be able to move the cervix gently from side to side, and doing so should produce no pain for the patient. Hardness of the cervix may occur with malignancy. Immobility may occur with malignancy, and pain may occur with inflammation or ectopic pregnancy.

A patient contacts the office and tells the nurse that she is worried about her 10-year-old daughter having breast cancer. She describes a unilateral enlargement of the right breast with associated tenderness. She is worried because the left breast is not enlarged. What would be the nurse's best response? A) Tell the mother that breast development is usually fairly symmetric and she should be examined right away. B) Tell the mother that she should bring her daughter in right away because breast cancer is fairly common in preadolescent girls. C) Tell the mother that, although an examination of her daughter would rule out a problem, it is most likely normal breast development. D) Tell the mother that it is unusual for breasts that are first developing to feel tender because they haven't developed much fibrous tissue.

ANS: C Occasionally one breast may grow faster than the other, producing a temporary asymmetry. This may cause some distress; reassurance is necessary. Tenderness is common also.

A 54-year-old man comes to the clinic with a "horrible problem." He tells the nurse that he has just discovered a lump on his breast and is fearful of cancer. The nurse knows that which statement about breast cancer in males is true? A) Breast masses in men are difficult to detect because of minimal breast tissue. B) Breast cancer in men rarely spreads to the lymph nodes. C) One percent of all breast cancer occurs in men. D) Most breast masses in men are diagnosed as gynecomastia.

ANS: C One percent of all breast cancer occurs in men. Early spread to axillary lymph nodes occurs due to minimal breast tissue.

A woman states that 2 weeks ago she had a urinary tract infection that was treated with an antibiotic. As a part of the interview, the nurse should ask, "Have you noticed: A) a change in your urination patterns?" B) any excessive vaginal bleeding?" C) any unusual vaginal discharge or itching?" D) any changes in your desire for intercourse?"

ANS: C Several medications may increase the risk of vaginitis. Broad-spectrum antibiotics alter the balance of normal flora, which may lead to the development of vaginitis. The other questions are not correct

The nurse notices that a patient has had a pale, yellow, greasy stool, or steatorrhea, and recalls that this is caused by: A) occult bleeding. B) absent bile pigment. C) increased fat content. D) ingestion of bismuth preparations.

ANS: C Steatorrhea (pale, yellow, greasy stool) is caused by increased fat content in the stools, as in malabsorption syndrome. Occult bleeding and ingestion of bismuth products cause black stool, and absent bile pigment causes gray, tan stool.

A 45-year-old mother of two children is seen at the clinic for complaints of "losing my urine when I sneeze." The nurse documents that she is experiencing: A) urinary frequency. B) enuresis. C) stress incontinence. D) urge incontinence.

ANS: C Stress incontinence is involuntary urine loss with physical strain, sneezing, or coughing that occurs due to weakness of the pelvic floor. Urinary frequency is urinating more times than usual (more than 5 to 6 times per day). Enuresis is involuntary passage of urine at night after age 5 to 6 years (bed wetting). Urge incontinence is involuntary urine loss from overactive detrusor muscle in the bladder. It contracts, causing an urgent need to void.

When the nurse is performing a testicular examination on a 25-year-old man, which of these findings is considered normal? A) Nontender subcutaneous plaques B) A scrotal area that is dry, scaly, and nodular C) Testes that feel oval and movable and are slightly sensitive to compression D) A single, hard, circumscribed, movable mass, less than 1 cm under the surface of the testes

ANS: C Testes normally feel oval, firm and rubbery, smooth, and equal bilaterally and are freely movable and slightly tender to moderate pressure. The scrotal skin should not be dry, scaly, or nodular or contain subcutaneous plaques. Any mass would be an abnormal finding.

During a discussion about breast self-examination with a 30-year-old woman, which of these statements by the nurse is most appropriate? A) "The best time to examine your breasts is during ovulation." B) "Examine your breasts every month on the same day of the month." C) "Examine your breasts shortly after your menstrual period each month." D) "The best time to examine your breasts is immediately before menstruation."

ANS: C The best time to conduct breast self-examination is shortly after the menstrual period when the breasts are the smallest and least congested.

In performing an assessment of a woman's axillary lymph system, the nurse should assess which of these nodes? A) Central, axillary, lateral, and sternal nodes B) Pectoral, lateral, anterior, and sternal nodes C) Central, lateral, pectoral, and subscapular nodes D) Lateral, pectoral, axillary, and suprascapular nodes

ANS: C The breast has extensive lymphatic drainage. Four groups of axillary nodes are present: (1) central, (2) pectoral (anterior), (3) subscapular (posterior), and (4) lateral.

The nurse is preparing for an internal genitalia examination of a woman. Which order of the examination is correct? A) Bimanual, speculum, rectovaginal B) Speculum, rectovaginal, bimanual C) Speculum, bimanual, rectovaginal D) Rectovaginal, bimanual, speculum

ANS: C The correct sequence is speculum examination, then bimanual examination after removing the speculum, and then rectovaginal examination. The examiner should change gloves before performing the rectovaginal examination to avoid spreading any possible infection

During an internal examination of a woman's genitalia, the nurse will use which technique for proper insertion of the speculum? A) Instruct the woman to bear down, open the speculum blades, and apply in a swift, upward movement. B) Insert the blades of the speculum on a horizontal plane, turning them to a 30-degree angle while continuing to insert them. Ask the woman to bear down after the speculum is inserted. C) Instruct the woman to bear down, turn the width of the blades horizontally, and insert the speculum at a 45-degree angle downward toward the small of the woman's back. D) Lock the blades open by turning the thumbscrew. Once the blades are open, apply pressure to the introitus and insert the blades at a 45-degree angle downward to bring the cervix into view.

ANS: C The examiner should instruct the woman to bear down, turn the width of the blades horizontally, and insert the speculum at a 45-degree angle downward toward the small of the woman's back. See the text under "Speculum Examination" for more detail.

A 55-year-old postmenopausal woman is being seen in the clinic for a yearly examination. She is concerned about changes in her breasts that she has noticed over the past 5 years. She states that her breasts have decreased in size and that the elasticity has changed so that her breasts seem "flat and flabby." The nurse's best reply would be: A) "This change occurs most often because of long-term use of bras that do not provide enough support to the breast tissues." B) "This is a normal change that occurs as women get older. It is due to the increased levels of progesterone during the aging process." C) "Decreases in hormones after menopause causes atrophy of the glandular tissue in the breast. This is a normal process of aging." D) "Postural changes in the spine make it appear that your breasts have changed in shape. Exercises to strengthen the muscles of the upper back and chest wall will help to prevent the changes in elasticity and size."

ANS: C The hormonal changes of menopause cause the breast glandular tissue to atrophy, making the breasts more pendulous, flattened, and sagging.

Which statement would be most appropriate when the nurse is introducing the topic of sexual relationships during an interview? A) "Now it is time to talk about your sexual history. When did you first have intercourse?" B) "Women often feel dissatisfied with their sexual relationships. Would it be okay to discuss this now?" C) "Often women have questions about their sexual relationship and how it affects their health. Do you have any questions?" D) "Most women your age have had more than one sexual partner. How many would you say you have had?"

ANS: C The nurse should begin with an open-ended question to assess individual needs. The nurse should include appropriate questions as a routine part of the history, because doing so communicates that the nurse accepts the individual's sexual activity and believes it is important. The nurse's comfort with discussion prompts the patient's interest and possibly relief that the topic has been introduced. This establishes a database for comparison with any future sexual activities and provides an opportunity to screen sexual problems.

During the physical examination, the nurse notices that a female patient has an inverted left nipple. Which statement regarding this is most accurate? A) Normal nipple inversion is usually bilateral. B) A unilateral inversion of a nipple is always a serious sign. C) It should be determined whether the inversion is a recent change. D) Nipple inversion is not significant unless accompanied by an underlying palpable mass.

ANS: C The nurse should distinguish a recently retracted nipple from one that has been inverted for many years or since puberty. Normal nipple inversion may be unilateral or bilateral and usually can be pulled out (i.e., it is not fixed). Recent nipple retraction signifies acquired disease. See Table 17-3.

The nurse is conducting a class about breast self-examination (BSE). Which of these statements indicates proper BSE technique? A) The best time to perform BSE is in the middle of the menstrual cycle. B) The woman needs to do BSE only bimonthly unless she has fibrocystic breast tissue. C) The best time to perform BSE is 4 to 7 days after the first day of the menstrual period. D) If she suspects that she is pregnant, the woman should not perform a BSE until her baby is born.

ANS: C The nurse should help each woman establish a regular schedule of self-care. The best time to conduct breast self-examination is right after the menstrual period, or the fourth through seventh day of the menstrual cycle, when the breasts are the smallest and least congested. Advise the pregnant or menopausal woman who is not having menstrual periods to select a familiar date to examine her breasts each month, for example, her birth date or the day the rent is due.

During an examination of a woman, the nurse notices that her left breast is slightly larger than her right breast. Which of these statements is true about this finding? A) Breasts should always be symmetric. B) This finding is probably due to breastfeeding and is nothing to worry about. C) This finding is not unusual, but the nurse should verify that this change is not new. D) This finding is very unusual and means she may have an inflammation or growth.

ANS: C The nurse should notice symmetry of size and shape. It is common to have a slight asymmetry in size; often the left breast is slightly larger than the right. A sudden increase in the size of one breast signifies inflammation or new growth.

The nurse is examining only the rectal area of a woman and should place the woman in what position? A) Lithotomy position B) Prone position C) Left lateral decubitus position D) Bending over the table while standing

ANS: C The nurse should place the female patient in lithotomy position if examining genitalia as well; use the left lateral decubitus position for the rectal area alone.

The nurse is preparing to teach a woman about breast self-examination (BSE). Which statement by the nurse is correct? A) "BSE is more important than ever for you because you have never had any children." B) "BSE is so important because one out of nine women will develop breast cancer in her lifetime." C) "BSE on a monthly basis will help you feel familiar with your own breasts and their normal variations." D) "BSE will save your life because you are likely to find a cancerous lump between mammograms."

ANS: C The nurse should stress that a regular monthly self-examination will familiarize her with her own breasts and their normal variations. This is a positive step that will reassure her of her healthy state. While teaching, the nurse should focus on the positive aspects of breast self-examination and should avoid citing frightening mortality statistics about breast cancer. This may generate excessive fear and denial that actually obstructs a woman's self-care action.

While assessing a hospitalized, bedridden patient, the nurse notices that the patient has been incontinent of stool. The stool is loose and gray-tan in color. The nurse recognizes that this finding indicates which of the following? A) Occult blood B) Inflammation C) Absent bile pigment D) Ingestion of iron preparations

ANS: C The presence of gray, tan stool indicates absent bile pigment, which can occur with obstructive jaundice. Ingestion of iron preparations and presence of occult blood would turn the stools to a black color. Jelly-like mucus shreds mixed in the stool would indicate inflammation.

A 46-year-old man requires assessment of his sigmoid colon. The nurse is aware that which of these is most appropriate for this examination? A) Proctoscope B) Ultrasound C) Colonoscope D) Rectal exam with an examining finger

ANS: C The sigmoid colon is 40 cm long and is accessible to examination only with the colonoscope. The other responses are not appropriate for examination of the entire sigmoid colon.

During a history interview, a female patient states that she has noticed a few drops of clear discharge from her right nipple. What should the nurse do next? A) Contact the physician immediately to report the discharge. B) Ask her if she is possibly pregnant. C) Ask her some additional questions about the medications she is taking. D) Immediately obtain a sample for culture and sensitivity testing.

ANS: C The use of some medications, such as oral contraceptives, phenothiazines, diuretics, digitalis, steroids, methyldopa, and calcium channel blockers, may cause clear nipple discharge. Bloody or blood-tinged discharge from the nipple, not clear, is significant, especially if a lump is also present. In the pregnant female, colostrum would be a thick, yellowish liquid, and it would be expressed after the fourth month of pregnancy.

While examining a 75-year-old woman, the nurse notices that the skin over her right breast is thickened and the hair follicles are exaggerated. This condition is known as: A) dimpling. B) retraction. C) peau d'orange. D) benign breast disease.

ANS: C This condition is known as peau d'orange. Lymphatic obstruction produces edema, which thickens the skin and exaggerates the hair follicles. The skin has a pig-skin or orange-peel look, and this condition suggests cancer.

During an examination, the nurse notices that a male patient has a red, round, superficial ulcer with a yellowish serous discharge on his penis. On palpation, the nurse finds a nontender base that feels like a small button between the thumb and fingers. At this point the nurse suspects that this patient has: A) genital warts. B) a herpes infection. C) a syphilitic chancre. D) a carcinoma lesion.

ANS: C This lesion indicates syphilitic chancre, which begins within 2 to 4 weeks of infection. See Table 24-4 for descriptions of the other options.

During an assessment of the newborn, the nurse expects to see which finding when the anal area is slightly stroked? A) A jerking of the legs B) Flexion of the knees C) A quick contraction of the sphincter D) Relaxation of the external sphincter

ANS: C To assess sphincter tone, the nurse should check the anal reflex by gently stroking the anal area and noticing a quick contraction of the sphincter. The other responses are not correct.

When the nurse is interviewing a preadolescent girl, which opening statement would be least threatening? A) "Do you have any questions about growing up?" B) "What has your mother told you about growing up?" C) "When did you notice that your body was changing?" D) "I remember being very scared when I got my period. How do you think you'll feel?"

ANS: C Try the open-ended, "When did you ... ?" rather than "Do you ... ?" This is less threatening because it implies that the topic is normal and unexceptional.

An 18-year-old patient is having her first pelvic examination. Which action by the nurse is appropriate? A) Invite her mother to be present during the examination. B) Avoid the lithotomy position this first time because it can be uncomfortable and embarrassing. C) Raise the head of the examination table and give her a mirror so that she can view the exam. D) Drape her fully, leaving the drape between her legs elevated to avoid embarrassing her with eye contact.

ANS: C Use the techniques of the educational or mirror pelvic examination. This is a routine examination with some modifications in attitude, position, and communication. First, the woman is considered an active participant, one who is interested in learning and in sharing decisions about her own health care. The woman props herself up on one elbow, or the head of the table is raised. Her other hand holds a mirror between her legs, above the examiner's hands. The woman can see all that the examiner is doing and has a full view of her genitalia. The mirror works well for teaching normal anatomy and its relationship to sexual behavior. You can ask her if she would like to have a family member, friend, or chaperone present for the examination. The drape should be pushed down between the woman's legs so that the nurse can see her face.

When a breastfeeding mother is diagnosed with a breast abscess, which of these instructions from the nurse is correct? The mother needs to: A) continue to nurse on both sides to encourage milk flow. B) discontinue nursing immediately to allow for healing. C) temporarily discontinue nursing on affected breast and manually express milk and discard it. D) temporarily discontinue nursing on affected breast but can manually express milk and give it to the baby.

ANS: C With a breast abscess, the patient must temporarily discontinue nursing on the affected breast, manually express the milk, and discard it. Nursing can continue on the unaffected side.

A 46-year-old woman is in the clinic for her annual gynecologic examination. She voices a concern about ovarian cancer because her mother and sister died of it. The nurse knows that which of these statements is correct regarding ovarian cancer? A) Ovarian cancer rarely has any symptoms. B) The Pap smear detects the presence of ovarian cancer. C) Women at high risk for ovarian cancer should have annual transvaginal ultrasonography for screening. D) Women over age 40 years should have a thorough pelvic examination every 3 years.

ANS: C With ovarian cancer, the patient may have abdominal pain, pelvic pain, increased abdominal size, bloating, and nonspecific gastrointestinal symptoms, or she may be asymptomatic. The Pap smear does not detect the presence of ovarian cancer. Annual transvaginal ultrasonography may detect ovarian cancer at an earlier stage in women who are at high risk for it.

The nurse knows that a common assessment finding in a boy younger than 2 years old is: A) an inflamed and tender spermatic cord. B) the presence of a hernia in the scrotum. C) a penis that looks large in relation to the scrotum. D) the presence of a hydrocele, or fluid in the scrotum.

ANS: D A common scrotal finding in boys younger than 2 years of age is a hydrocele, or fluid in the scrotum. The other options are not correct.

While performing a rectal examination, the nurse notices a firm, irregularly shaped mass. What should the nurse do next? A) Continue with the examination and document the finding in the chart. B) Instruct patient to return for repeat assessment in 1 month. C) Tell the patient that a mass was felt but it is nothing to worry about. D) Report the finding and refer the patient to a specialist for further examination.

ANS: D A firm or hard mass with irregular shape or rolled edges may signify carcinoma. Promptly report any mass that is discovered for further examination. The other responses are not correct

A 62-year-old man states that his doctor told him that he has an "inguinal hernia." He asks the nurse to explain what a hernia is. The nurse should: A) tell him not to worry and that most men his age develop hernias. B) explain that a hernia is often the result of prenatal growth abnormalities. C) refer him to his physician for additional consultation because the physician made the initial diagnosis. D) explain that a hernia is a loop of bowel protruding through a weak spot in the abdominal muscles.

ANS: D A hernia is a loop of bowel protruding through a weak spot in the musculature. The other options are not correct responses to the patient's question.

During an assessment of a 20-year-old man, the nurse finds a small palpable lesion with a tuft of hair located directly over the coccyx. The nurse knows that this lesion would most likely be a: A) polyp. B) pruritus ani. C) carcinoma. D) pilonidal cyst.

ANS: D A pilonidal cyst or sinus is a hair-containing cyst or sinus located in the midline over the coccyx or lower sacrum. It often opens as a dimple with a visible tuft of hair and, possibly, an erythematous halo. See Table 25-1 for more information, and also for description of pruritus ani. See Table 25-2 for descriptions of rectal polyps and carcinoma.

Flexion

Bending a limb at a joint

A 15-year-old boy is seen in the clinic for complaints of "dull pain and pulling" in the scrotal area. On examination the nurse palpates a soft, irregular mass posterior to and above the testis on the left. This mass collapses when the patient is supine and refills when he is upright. This description is consistent with: A) epididymitis. B) spermatocele. C) testicular torsion. D) varicocele.

ANS: D A varicocele consists of dilated, tortuous varicose veins in the spermatic cord caused by incompetent valves within the vein. Symptoms include dull pain or constant pulling or dragging feeling, or the individual may be asymptomatic. When palpating the mass, the examiner will feel a soft, irregular mass posterior to and above the testis that collapses when the individual is supine and refills when the individual is upright. See Table 24-6 for more information and for descriptions of the other options.

A 9-year-old girl is in the clinic for a sports physical. After some initial shyness she finally asks, "Am I normal? I don't seem to need a bra yet, but I have some friends who do. What if I never get breasts?" The nurse's best response would be: A) "Don't worry, you still have plenty of time to develop." B) "I know just how you feel, I was a late bloomer myself. Just be patient and they will grow." C) "You will probably get your periods before you notice any significant growth in your breasts." D) "I understand that it is hard to feel different from your friends. Breasts usually develop between 8 and 10 years of age."

ANS: D Adolescent breast development usually begins between 8 and 10 years of age. The nurse should not belittle the girl's feelings by using statements like "don't worry" or by sharing personal experiences. The beginning of breast development precedes menarche by about 2 years.

A 65-year-old patient remarks that she just can't believe that her breasts sag so much. She states it must be from lack of exercise. What explanation should the nurse offer her? A) After menopause, only women with large breasts experience sagging. B) After menopause, sagging is usually due to decreased muscle mass within the breast. C) After menopause, a diet that is high in protein will help maintain muscle mass, which keeps the breasts from sagging. D) After menopause, the glandular and fat tissue atrophies, causing breast size and elasticity to diminish, resulting in breasts that sag.

ANS: D After menopause, the glandular tissue atrophies and is replaced with connective tissue. The fat envelope atrophies also, beginning in the middle years and becoming marked in the eighth and ninth decades. These changes decrease breast size and elasticity, so the breasts droop and sag, looking flattened and flabby

The nurse is teaching a pregnant woman about breast milk. Which statement by the nurse is correct? A) "Your breast milk is present immediately after delivery of the baby." B) "Breast milk is rich in protein and sugars (lactose) but has very little fat." C) "The colostrum, which is present right after birth, does not contain the same nutrition as breast milk does." D) "You may notice a thick, yellow fluid expressed from your breasts as early as the fourth month of pregnancy."

ANS: D After the fourth month, colostrum may be expressed. This thick yellow fluid is the precursor of milk, and it contains the same amount of protein and lactose but practically no fat. The breasts produce colostrum for the first few days after delivery. It is rich with antibodies that protect the newborn against infection, so breastfeeding is importat

Which of these statements is true regarding the penis? A) The urethral meatus is located on the ventral side of the penis. B) The prepuce is the fold of foreskin covering the shaft of the penis. C) The penis is composed of two cylindrical columns of erectile tissue. D) The corpus spongiosum expands into a cone of erectile tissue called the glans.

ANS: D At the distal end of the shaft, the corpus spongiosum expands into a cone of erectile tissue, the glans. The penis is composed of three cylindrical columns of erectile tissue. The prepuce is skin that covers the glans of the penis. The urethral meatus forms at the tip of the glans.

A 35-year-old woman is at the clinic for a gynecologic examination. During the examination, she asks the nurse, "How often do I need to have this Pap test done?" Which reply by the nurse is correct? A) "It depends. Do you smoke?" B) "This will need to be done annually until you are 65." C) "If you have 2 consecutive normal Pap tests, then you can wait 5 years between tests." D) "After age 30, if you have 3 consecutive normal Pap tests, then you may be screened every 2 to 3 years."

ANS: D Cervical cancer screening with the Pap test continues annually until age 30. After age 30, if the woman has 3 consecutive normal Pap tests, then women may be screened every 2 to 3 years.

The nurse is performing a breast examination. Which of these statements best describes the correct procedure to use when screening for nipple and skin retraction during a breast examination? Have the woman: A) bend over and touch her toes. B) lie down on her left side and notice any retraction. C) shift from a supine position to a standing position; notice any lag or retraction. D) slowly lift her arms above her head and note any retraction or lag in movement.

ANS: D Direct the woman to change position while checking the breasts for skin retraction signs. First ask her to lift her arms slowly over her head. Both breasts should move up symmetrically. Retraction signs are due to fibrosis in the breast tissue, usually caused by growing neoplasms. The nurse should notice if there is a lag in movement of one breast.

A 22-year-old woman is being seen at the clinic for problems with vulvar pain, dysuria, and fever. On physical examination, the nurse notices clusters of small, shallow vesicles with surrounding erythema on the labia. There is also inguinal lymphadenopathy present. The most likely cause of these lesions is: A) pediculosis pubis. B) contact dermatitis. C) human papillomavirus. D) herpes simplex virus type 2.

ANS: D Herpes simplex virus type 2 presents with clusters of small, shallow vesicles with surrounding erythema that erupt on the genital areas. There is also the presence of inguinal lymphadenopathy. The individual reports local pain, dysuria, and fever. See Table 26-2 for more information and descriptions of the other conditions.

During a health history, a patient tells the nurse that he has trouble in starting his urine stream. This problem is known as: A) urgency. B) dribbling. C) frequency. D) hesitancy.

ANS: D Hesitancy is trouble in starting the urine stream. Urgency is the feeling that one cannot wait to urinate. Dribbling is the lost of urine before or after the main act of urination. Frequency is urinating more often than usual.

The nurse is performing a genital examination on a male patient and notices urethral drainage. When collecting urethral discharge for microscopic examination and culture, the nurse should: A) ask the patient to urinate into a sterile cup. B) ask the patient to obtain a specimen of semen. C) insert a cotton-tipped applicator into the urethra. D) compress the glans between the examiner's thumb and forefinger and collect any discharge.

ANS: D If urethral discharge is noticed, then the examiner should collect a smear for microscopic examination and culture by compressing the glans anteroposteriorly between the thumb and forefinger. The other options are not correct actions

Which of these clinical situations would the nurse consider to be outside normal limits? A) A patient has had one pregnancy. She states that she believes she may be entering menopause. Her breast examination reveals breasts that are soft and sag slightly. B) A patient has never been pregnant. Her breast examination reveals large pendulous breasts that have a firm, transverse ridge along the lower quadrant in both breasts. C) A patient has never been pregnant. She reports that she should begin her period tomorrow. Her breast examination reveals breast tissue that is nodular and somewhat engorged. She states that the examination was slightly painful. D) A patient has had two pregnancies and she breastfed both of her children. Her youngest child is now 10 years old. Her breast examination reveals breast tissue that is somewhat soft and she has a small amount of thick yellow discharge from both nipples.

ANS: D In nulliparous women, normal breast tissue feels firm, smooth, and elastic; after pregnancy, the tissue feels softer and looser. If any discharge appears, the nurse should note its color and consistency. Except in pregnancy and lactation, discharge is abnormal. Premenstrual engorgement is normal, and consists of a slight enlargement, tenderness to palpation, and a generalized nodularity. A firm, transverse ridge of compressed tissue in the lower quadrants, known as the inframammary ridge, is especially noticeable in large breasts.

During an examination, the nurse asks the patient to perform the Valsalva maneuver and notices that the patient has a moist, red, doughnut-shaped protrusion from the anus. The nurse knows that this would be consistent with: A) a rectal polyp. B) hemorrhoids. C) a rectal fissure. D) rectal prolapse.

ANS: D In rectal prolapse, the rectal mucous membrane protrudes through the anus, appearing as a moist red doughnut with radiating lines. It occurs following a Valsalva maneuver, such as straining at stool, or with exercise. See Table 25-1. For a description of rectal polyps, see Table 25-2. See Table 25-1 for descriptions of rectal fissure and hemorrhoids.

When the nurse is conducting sexual history from a male adolescent, which statement would be most appropriate to use at the beginning of the interview? A) "Do you use condoms?" B) "You don't masturbate, do you?" C) "Have you had sex in the last 6 months?" D) "Often boys your age have questions about sexual activity."

ANS: D Start the interview with a permission statement. This conveys that it is normal and all right to think or feel a certain way. Avoid sounding judgmental.

When the nurse is performing a genital examination on a male patient, the patient has an erection. The nurse's most appropriate action or response is to: A) ask the patient if he would like someone else to examine him. B) continue with the examination as though nothing has happened. C) stop the examination, leave the room while stating that the examination will resume at a later time. D) reassure the patient that this is a normal response and continue with the examination.

ANS: D When the male patient has an erection, the nurse should reassure the patient that this is a normal physiologic response to touch and proceed with the rest of the examination. The other responses are not correct and may be perceived as judgmental.

A 65-year-old woman is in the office for routine gynecologic care. She had a complete hysterectomy 3 months ago after cervical cancer was detected. The nurse knows that which of these statements is true with regard to this visit? A) Her cervical mucosa will be red and dry looking. B) She will not need to have a Pap smear done. C) The nurse can expect to find that her uterus will be somewhat enlarged and her ovaries small and hard. D) The nurse should plan to lubricate the instruments and the examining hand well to avoid a painful examination.

ANS: D In the aging adult woman, natural lubrication is decreased; to avoid a painful examination, the nurse should take care to lubricate instruments and the examining hand adequately. Menopause, with the resulting decrease in estrogen production, shows numerous physical changes. The cervix shrinks and looks pale and glistening. With the bimanual examination, the uterus feels smaller and firmer and the ovaries are not palpable normally. Women should continue cervical cancer screening up to age 70 years if they have an intact cervix and are in good health. Women who have had a total hysterectomy for benign gynecologic disease do not need cervical cancer screening, but if the hysterectomy was done for cervical cancer, then Pap tests should continue until the patient has a 10-year history of no abnormal results.

A 13-year-old girl is visiting the clinic for a sports physical. The nurse should remember to include which of these tests in the examination? A) Test for occult blood B) The Valsalva maneuver C) Internal palpation of the anus D) Inspection of the perianal area

ANS: D Inspect the perianal region of the school-aged child and adolescent during examination of the genitalia. Internal palpation is not performed routinely at this age. Testing for occult blood and doing the Valsalva maneuver are also not necessary.

A 22-year-old woman has been considering using oral contraceptives. As a part of her history, the nurse should ask: A) "Do you have a history of heart murmurs?" B) "Will you be in a monogamous relationship?" C) "Have you thought this choice through carefully?" D) "If you smoke, how many cigarettes do you smoke per day?"

ANS: D Oral contraceptives, together with cigarette smoking, increase the risk for cardiovascular side effects. If cigarettes are used, then the nurse should assess smoking history. The other questions are not appropriate.

The nurse is providing patient teaching about an erectile dysfunction drug. One of the drug's potential side effects is prolonged, painful erection of the penis without sexual stimulation, which is known as: A) orchitis. B) stricture. C) phimosis. D) priapism.

ANS: D Priapism is prolonged, painful erection of the penis without sexual desire. Orchitis is inflammation of the testes. Stricture is a narrowing of the opening of the urethral meatus. Phimosis is the inability to retract the foreskin

The nurse is reviewing risk factors for breast cancer. Which of these women have risk factors that place them at a higher risk for breast cancer? A) 37 year old who is slightly overweight B) 42 year old who has had ovarian cancer C) 45 year old who has never been pregnant D) 65 year old whose mother had breast cancer

ANS: D Risk factors for breast cancer include having a first-degree relative with breast cancer (mother, sister, or daughter) and being older than 50 years. Refer to Table 17- 2 for other risk factors.

A woman who is 8 weeks pregnant is in the clinic for a checkup. The nurse reads on her chart that her cervix is softened and looks cyanotic. The nurse knows that the woman is exhibiting _____ sign and _____ sign. A) Tanner's; Hegar's B) Hegar's; Goodell's C) Chadwick's; Hegar's D) Goodell's; Chadwick's

ANS: D Shortly after the first missed menstrual period, the female genitalia show signs of the growing fetus. The cervix softens (Goodell's sign) at 4 to 6 weeks, and the vaginal mucosa and cervix look cyanotic (Chadwick's sign) at 8 to 12 weeks. These changes occur because of increased vascularity and edema of the cervix and hypertrophy and hyperplasia of the cervical glands. Hegar's sign occurs when the isthmus of the uterus softens at 6 to 8 weeks. Tanner's sign is not a correct response.

During a health history, a 22-year old woman asks, "Can I get that vaccine for HPV? I have genital warts and I'd like them to go away!" What is the nurse's best response? A) "The HPV vaccine is for girls and women ages 9 to 26, so we can start that today." B) "This vaccine is only for girls who have not started to have intercourse yet." C) "Let's check with the physician to see if you are a candidate for this vaccine." D) "The vaccine cannot protect you if you already have an HPV infection."

ANS: D The HPV (human papillomavirus) vaccine is appropriate for girls and women age 9 to 26 and is given to prevent cervical cancer by preventing HPV infections before girls become sexually active. However, it cannot protect the woman if an HPV infection is already present.

Which of these statements about the anal canal is true? The anal canal: A) is about 2 cm long in the adult. B) slants backward toward the sacrum. C) contains hair and sebaceous glands. D) is the outlet for the gastrointestinal tract.

ANS: D The anal canal is the outlet for the gastrointestinal tract, and it is about 3.8 cm long in the adult. It is lined with a modified skin that does not contain hair or sebaceous glands, and it slants forward toward the umbilicus.

While doing an assessment of the perianal area of a patient, the nurse notices that the pigmentation of anus is darker than surrounding skin, the anal opening is closed, and there is a skin sac that is shiny and blue. The patient mentioned that he has had pain with bowel movements and has noted some spots of blood occasionally. What would this assessment and history be most likely to indicate? A) Anal fistula B) Pilonidal cyst C) Rectal prolapse D) Thrombosed hemorrhoid

ANS: D The anus normally looks moist and hairless, with coarse folded skin that is more pigmented than the perianal skin. The anal opening is tightly closed. The shiny blue skin sac indicates a thrombosed hemorrhoid.

The nurse is examining the glans and knows that which of these is a normal finding for this area? A) The meatus may have a slight discharge when the glans is compressed. B) Hair is without pest inhabitants. C) The skin is wrinkled and without lesions. D) Smegma may be present under the foreskin of an uncircumcised male.

ANS: D The glans looks smooth and without lesions and does not have hair. The meatus should not have any discharge when the glans is compressed. Some cheesy smegma may have collected under the foreskin of an uncircumcised male.

During the assessment of an 18-month-old child, the mother expresses concern to the nurse about the infant's inability to toilet train. What would be the nurse's best response? A) "Some children are just more difficult to train, so I wouldn't worry about it yet." B) "Have you considered reading any of the books on toilet training? They can be very helpful." C) "This could mean there is a problem in your baby's development. We'll watch her closely for the next few months." D) "The nerves that will allow your baby to have control over the passing of stools are not developed until at least 18 to 24 months of age."

ANS: D The infant passes stools by reflex. Voluntary control of the external anal sphincter cannot occur until the nerves supplying the area have become fully myelinated, usually around 1 1/2 to 2 years of age. Toilet training usually starts after the age of 2.

When performing a genital assessment on a middle-aged man, the nurse notices multiple soft, moist, painless papules in the shape of cauliflower-like patches scattered across the shaft of the penis. These lesions are characteristic of: A) carcinoma. B) syphilitic chancres. C) genital herpes. D) genital warts.

ANS: D The lesions of genital warts are soft, pointed, moist, fleshy, painless papules that may be single or multiple in a cauliflower-like patch. They occur on the shaft of the penis, behind the corona, or around the anus, where they may grow into large grape-like clusters. See Table 24-4 for more information and for descriptions of the other options.

A 60-year-old man has just been told he has benign prostatic hypertrophy. He has a friend who just died from cancer of the prostate, and he is concerned this will happen to him. How should the nurse respond? A) "The swelling in your prostate is only temporary and will go away." B) "We will treat you with chemotherapy so we can control the cancer." C) "It would be very unusual for a man your age to have cancer of the prostate." D) "The enlargement of your prostate is caused by hormone changes and not cancer."

ANS: D The prostate gland commonly starts to enlarge during the middle adult years. This benign prostatic hypertrophy (BPH) is present in 1 out of 10 males at the age of 40 years and increases with age. It is thought that the hypertrophy is caused by hormonal imbalance that leads to the proliferation of benign adenomas. The other responses are not appropriate.

A 25-year-old woman comes to the emergency department with a sudden fever of 101° F and abdominal pain. Upon examination, the nurse notices that she has rigid, boardlike lower abdominal musculature. When the nurse tries to perform a vaginal examination, the patient has severe pain when the uterus and cervix are moved. The nurse knows that these signs and symptoms are suggestive of: A) endometriosis. B) uterine fibroids. C) ectopic pregnancy. D) pelvic inflammatory disease.

ANS: D These signs and symptoms are suggestive of acute pelvic inflammatory disease, also known as acute salpingitis. See Table 26-7. For description of endometriosis and uterine fibroids, see Table 26-6; for description of ectopic pregnancy, see Table 26-7.

During a history, the patient states, "It really hurts back there, and sometimes it itches, too. I have even seen blood on the tissue when I have a bowel movement. Is there something there?" The nurse should expect to see which of these upon examination of the anus? A) Rectal prolapse B) Internal hemorrhoid C) External hemorrhoid that has resolved D) External hemorrhoid that is thrombosed

ANS: D These symptoms are consistent with an external hemorrhoid. An external hemorrhoid, when thrombosed, contains clotted blood and becomes a painful, swollen, shiny blue mass that itches and bleeds with defecation. When the external hemorrhoid resolves, it leaves a flabby, painless skin sac around the anal orifice. An internal hemorrhoid is not palpable, but may appear as a red mucosal mass when the person performs a Valsalva maneuver. A rectal prolapse appears as a moist, red doughnut with radiating lines

Which of these statements about the testes is true? A) The lymphatics of the testes drain into the abdominal lymph nodes. B) The vas deferens is located along the inferior portion of each testis. C) The right testis is lower than the left because the right spermatic cord is longer. D) The cremaster muscle contracts in response to cold and draws the testicles closer to the body.

ANS: D When it is cold, the cremaster muscle contracts, which raises the scrotal sac and brings the testes closer to the body to absorb heat necessary for sperm viability. The lymphatics of the testes drain into the inguinal lymph nodes. The vas deferens is located along the upper portion of each testis. The left testis is lower than the right because the left spermatic cord is longer.

When the nurse is performing a genital examination on a male patient, which of these actions is correct? A) Auscultate for the presence of a bruit over the scrotum. B) Palpate for the vertical chain of lymph nodes along the groin inferior to the inguinal ligament. C) Palpate the inguinal canal only if there is a bulge present in the inguinal region during inspection. D) Have the patient shift his weight onto the left (unexamined) leg when palpating for a hernia on the right side.

ANS: D When palpating for the presence of a hernia on the right side, ask the male patient to shift his weight onto the left (unexamined) leg. It is not appropriate to auscultate for a bruit over the scrotum. When palpating for lymph notes, palpate the horizontal chain. The inguinal canal should be palpated whether or not a bulge is present.

When testing stool for occult blood, the nurse is aware that a false-positive result may occur with: A) absent bile pigment. B) increased fat content. C) increased ingestion of iron medication. D) a large amount of red meat within the last 3 days.

ANS: D When testing for occult blood, a false-positive finding may occur if the person has ingested significant amounts of red meat within 3 days of the test. Absent bile pigment causes the stools to be gray or tan in color. Increased fat content causes the stool to be pale, yellow, and greasy. Increased ingestion of iron medication causes the stool to be black in color.

When performing a genitourinary assessment on a 16-year-old boy, the nurse notices a swelling in the scrotum that increases with increased intra-abdominal pressure and decreases when he is lying down. The patient complains of pain when straining. The nurse knows that this description is most consistent with a(n) _____ hernia. A) femoral B) incisional C) direct inguinal D) indirect inguinal

ANS: D With indirect inguinal hernias there is pain with straining and a soft swelling that increases with increased intra-abdominal pressure, which may decrease when the patient lies down. These findings do not describe the other hernias. See Table 24-7 for descriptions of femoral and direct inguinal hernias

When testing for muscle strength, the examiner should: A) observe muscles for the degree of contraction when the individual lifts a heavy object. B) apply an opposing force when the individual puts a joint in flexion or extension. C) measure the degree of force that it takes to overcome joint flexion or extension. D) estimate the degree of flexion and extension in each joint.

B) apply an opposing force when the individual puts a joint in flexion or extension.

An appropriate tool to assess an individual's instrumental activities of daily living is a tool by: A. Katz B. Lawton C. Tinetti D. Norbeck

B. Lawton

A patient requests to be discharged to home instead of a rehabilitation hospital after a hip fracture. Which of the following is true about the difference between home care and hospital care? A. Home care is more expensive than hospitalization. B. Patients have less risk for infection in the home setting. C. Patients have been shown to recover more slowly at home than in the hospital. D. Physical therapy is available only in the hospital setting.

B. Patients have less risk for infection in the home setting. RATIONALE: Older adults may avoid the risk of infection exposure when at home. Home care is less expensive than hospitalization. Older adults have been shown to recover more quickly when at home than when placed in an institution. Home care services include skilled nursing care; primary care; physical, occupational, and speech therapy; social work; nutrition; case management; assistance with activities of daily living; and some durable medical equipment.

The Get Up and Go Test would be used to: A. determine a patient's ability to get dressed without assistance. B. assess functional activity of the patient along with safety determination. C. assess swallowing status of the patient. D. assess adults with dementia.

B. assess functional activity of the patient along with safety determination. RATIONALE: The Get Up and Go Test is a reliable and valid test used to assess functional ability and safety aspects. Determination of functional assessment would be able to determine if the individual could get dressed without assistance. This test would not be used to evaluate a patient's swallowing status. The Direct Assessment of Functional Abilities assesses adults with dementia.

An older adult has had surgery for a fractured hip and has a history of dementia. You should keep in mind that older adults with cognitive impairment: A. experience less pain B. can provide a self-report of pain C. cannot be relied on to self-report pain D. will not express pain sensations

B. can provide a self-report of pain

Altered cognition in older adults is commonly attributed to: A. an infection or injury. B. dementia, delirium, or depression. C. the normal aging process. D. medication side effects.

B. dementia, delirium, or depression. RATIONALE: Altered cognition in older adults is commonly attributed to three disorders: dementia, delirium, and depression.

When completing a spiritual assessment, you should: A. use "yes" and "no" questions as the foundation for future dialogue B. use open-ended questions to help the patient understand potential coping mechanisms C. try to complete this assessment as soon as possible after meeting the patient D. wait until a member of the clergy can be involved in the assessment

B. use open-ended questions to help the patient understand potential coping mechanisms

The Katz Index of Independence in ADL would measure the functional ability to: A. clean the house and take out the garbage. B. wash the face and hands and comb hair. C. pay the electric and telephone bills. D. do laundry and put away the clothes.

B. wash the face and hands and comb hair. RATIONALE: The Katz Index of Independence in ADL is a functional assessment of a person's ability to complete activities of daily living (e.g., eating/feeding, bathing, grooming, dressing, toileting, walking, using stairs, and transferring). Cleaning the house and taking out the garbage are instrumental activities of daily living (e.g., abilities necessary for independent community living). Paying bills is an instrumental activity of daily living. Doing the laundry and putting away clothes are instrumental activities of daily living.

A pregnant woman who has breast implants asks the nurse if she can still breastfeed. What is the nurse's best response? A. "You should not have any problems breast feeding because your implants do not affect milk production." B. "When the breast implants are inserted they usually affect the milk glands, and breastfeeding is not possible." C. "This would depend on which type of implants were placed and which procedure was used by the surgeon. Check with your surgeon to see if your milk production will be affected."

C Milk production may vary depending on the procedure. Many women can still breastfeed after breast augmentation. The surgeon would be able to determine this ability. Answers 1 and 2 are incorrect because they may not be true. Only the surgeon can determine if milk production will be affected.

The divisions of the spinal vertebrae include: A) Cervical, thoracic, scaphoid, sacral, and clavicular. B) Scapular, clavicular, lumbar, scaphoid, and fasciculi. C) Cervical, thoracic, lumbar, sacral, and coccygeal. D) Cervical, lumbar, iliac, synovial, and capsular.

C) Cervical, thoracic, lumbar, sacral, and coccygeal.

Which is an example of a formal social support network for the aging adult? A. a neighbor who drops by with newspapers and magazines on a regular basis B. An area church that offers a weekly activity and luncheon for seniors in the neighborhood C. a home health care agency that provides weekly blood pressure screenings at the church luncheon D. a senior citizen chess club whose members hold classes at the local Boys' Club

C. a home health care agency that provides weekly blood pressure screenings at the church luncheon

An appropriate use of the caregiver strain index would be which situation? A. a daughter who is taking her older father home to live with her B. an older patient who lives alone C. a wife who has care for her husband for the past 4 years at home D. A son whose parents live in an assisted living facility

C. a wife who has care for her husband for the past 4 years at home

Which statement is true regarding an individual's functional status? A. functional status refers to one's ability to care for another person B. an older adult's functional status is usually static over time C. an older adult's functional status may vary from independence to disability D. dementia is an example of function status

C. an older adult's functional status may vary from independence to disability

When completing a health assessment of an older adult with mobility problems, the sequence should: A. begin with the physical examination followed by the health history. B. be from head to toe to prevent missing any important assessments. C. be arranged to minimize the number of position changes for the patient and the examiner. D. start with the most invasive assessments.

C. be arranged to minimize the number of position changes for the patient and the examiner. RATIONALE: If an older adult patient has limited mobility, the examiner should arrange the sequence to minimize the number of position changes for the patient. The health history should be collected before the physical examination. A head-to-toe approach may include numerous position changes and should be avoided for an older adult with mobility problems. Completing invasive assessments at the end of the examination decreases anxiety and embarrassment for an older adult patient.

Prevention and treatment of ____________ may be one of the most effective interventions aimed at reducing functional decline in an older adult. A. visual disturbances B. muscle weakness C. depression D. bladder and bowel incontinence

C. depression RATIONALE: Prevention and treatment of depression may be one of the most effective interventions aimed at reducing functional decline in an older adult.

Signs of caregiver burnout include: A. going to church every week. B. weight gain. C. headaches and epigastric pain. D. using an adult daycare facility.

C. headaches and epigastric pain. RATIONALE: Signs of possible caregiver burnout include multiple somatic complaints, increased stress and anxiety, social isolation, depression, and weight loss. Social isolation is a sign of caregiver burnout. Weight loss is a sign of caregiver burnout. Use of an adult daycare facility may prevent caregiver burnout.

It is dangerous for a cognitive change to be attributed to the normal aging process because: A. cognitive change is not associated with aging. B. nurses are not trained properly to make these types of judgments. C. this may delay the diagnosis of an underlying disease process. D. the client could be saying confusing comments to avoid detection of addictions.

C. this may delay the diagnosis of an underlying disease process. RATIONALE: Cognitive impairment resulting from disease may be attributed by patients, families, and health care providers to normal changes with aging, which can delay diagnostic workup.

Which of the following would be an indication that an older adult should stop driving a vehicle? A. Taking insulin to control type 2 diabetes mellitus B. Difficulty walking and getting in and out of the vehicle C. A pacemaker placed 2 months ago for complete heart block D. Difficulty checking over the shoulder when backing up or changing lanes

D. Difficulty checking over the shoulder when backing up or changing lanes RATIONALE: The American Association of Retired Persons has developed warning signs for when to stop driving. One of the warning signs is difficulty turning around to check over the shoulder while backing up or changing lanes. Type 2 diabetes mellitus and taking insulin are not indications to stop driving a vehicle. Mobility problems (e.g., difficulty walking or getting in or out of a vehicle) are not indications to stop driving a vehicle. Having a pacemaker is not an indication to stop driving a vehicle.

Which of the following would be included in an assessment of a patient's ability to perform instrumental activities of daily living? A. Balance, gait, and motor coordination B. Dressing, toileting, and using stairs C. Eating, bathing, and grooming D. Taking medications, shopping, and meal preparation

D. Taking medications, shopping, and meal preparation RATIONALE: Instrumental activities of daily living are functional abilities necessary for independent community living and include shopping, meal preparation, housekeeping, laundry, managing finances, taking medications, and using transportation. Other activities may include yard work or home maintenance and leisure activities (e.g., reading, other hobbies). Mobility or physical performance includes balance, gait, motor coordination, and endurance. Activities of daily living are tasks necessary for self-care and include eating/feeding, bathing, grooming, dressing, toileting, walking, using stairs, and transferring.

You will use which technique when assessing an older individual who has cognitive impairment? A. ask open-ended questions B. complete the entire assessment in one session C. ask the family members for information instead of the older individual D. ask simple questions that have "yes" or "no" answers

D. ask simple questions that have "yes" or "no" answers

Abduction

Moving a limb away from the midline of the body

Extension

Straightening a limb at a joint

Briefly describe the functions of the musculoskeletal system.

The musculoskeletal system provides support to the body, enabling it to stand erect and to move. The system protects inner organs, produces red blood cells, and provides for the storage of minerals.

This is the first visit for a woman, age 38 years. The practitioner instructs her that a baseline mammogram is recommended for women between the ages of 35 and 39 years and that the clinical examination schedule would be based on age. The recommendation for women ages 40 to 49 years is: a) every year b) every 2 years c) twice a year d) only the baseline examination is needed unless the woman has symptoms

a

Functional status

a person's actual performance of activities and task associated with current life roles

The bulge sign is a test for: a. swelling in the suprapatellar pouch. b. carpal tunnel syndrome c. Herberden's nodes d. olecranon bursa inflammation.

a. swelling in the suprapatellar pouch.

Examination of the shoulder includes four motions. These are: a.forward flexion, internal rotation, abduction, and external rotation. b. abduction, adduction, pronation, and supination. c. circumduction, inversion, eversion, and rotation. d. elevation, retraction, protraction, and circumduction.

a.forward flexion, internal rotation, abduction, and external rotation.

Advanced activities of daily living

activities that an older adult performs as a family member or as a member of society or community, including occupational and recreational activities

Lawton Instrumental Activities of Daily Living

an instrument used to measure an individual's ability to perform instrumental activities of daily living; may assist in assessing one's ability to live independently

Katz Index of independence in activities of daily living

an instrument used to measure physical function in older adults and the chronically ill

Environmental

assessment of an individual's home environment and community system, including hazards in the home

Spiritual assessment

assessment of an individual's spiritual health

Caregiver assessment

assessment of the health and well-being of an individual's caregiver

A 2-year-old child has been brought to the clinic for a health examination. A common finding would be: a) kyphosis b) lordosis c) scoliosis d) no deviation is normal

b

Breast asymmetry: a) increases with age and parity b) may be normal c) indicated a neoplasm d) is accompanied by enlarged axillary lymph nodes

b

During a breast examination, you detect a mass. Identify the description that is most consistent with cancer rather than benign breast disease.

b

During the examination of a 30-year-old woman, she questions you about the "2 large moles" that are below her left breast. After examining the area, how do you respond? a) "I think you should be examined by a dermatologist." b) "This appears to be a normal finding of supernumerary nipples, due to the small areolae and nipples that are present." c) "These are Montgomery's glands, which are common." d) "Is there a possibility you are pregnant?"

b

Pronation and supination of the hand and forearm are the result of the articulation of the: a) scapula and clavicle b) radius and ulna c) patella and condyle of fibula d) femur and acetabulum

b

The most common site of breast tumors is: a) upper inner quadrant b) upper outer quadrant c) lower inner quadrant d) lower outer quadrant

b

When assessing an infant, the examiner completes Ortolani maneuver by: a) lifting the newborn and noting a C-shaped curvature of the spine b) gently lifting and abducting the infant's flexed knees while palpating the great trochanter with the fingers c) comparing the height of the tops of the knees when the knees are flexed up d) palpating the length of the clavicles

b

Which is the first physical change associated with puberty in girls? a) areolar elevation b) breast bud development c) height spurt d) pubic hair development e) menarche

b

Pronation and supination of the hand and forearm are the result of the articulation of the : a. scapula and clavicle b. radius and ulna c. patella and condyle of fibula d. femur and acetabulum

b. radius and ulna

A 70-year-old woman has come for a health examination. Which of the following is a common age-related change in the curvature of the spinal column? a) lordosis b) scoliosis c) kyphosis d) lateral scoliosis

c

A bimanual technique may be the preferred approach for a woman: a) who is pregnant b) who is having the first breast examination by a health care provider c) with pendulous breasts d) who has felt a change in the breast during self-examination.

c

During the examination of a 70-year-old man, you note gynecomastia. You would: a) refer for a biopsy. b) refer for a mammogram. c) review the medications for drugs that have gynecomastia as a side effect. d) proceed with the examination. This is a normal part of the aging process.

c

Positive Phalen test and Tinel sign are seen in a patient with: a) a torn meniscus b) hallux valgus c) carpal tunnel syndrome d) tennis elbow

c

The breasts of a neonate may be large and very visible, secreting clear or white fluid. What is the basis of this finding? a) It may be due to birth trauma b) The fluid is colostrum, which is typically seen as a precursor to milk. c) The cause is maternal estrogen, which crossed the placenta. d) This often occurs with premature thelarche.

c

The examiner is going to inspect the breasts for retraction. The best position for this part of the examination is: a) lying supine with arms at the sides b) leaning forward with hands outstretched c) sitting with hand pushing onto hips d) one arm at the side, the other arm elevated

c

The examiner is going to measure the patient's legs for length discrepancy. The normal finding would be: a) no difference in measurements b) 0.5 cm difference c) within 1 cm of each other d) 2 cm difference

c

A 70-year-old woman has come for a health examination. Which of the following is a common age-related change in the curvature of the spinal column? a. lordosis b. scoliosis c. kyphosis d. lateral scoliosis

c. kyphosis

The examiner is going to measure the patient's legs for discrepancy. The normal finding would be: a. no difference in measurements. b. 0.5 cm difference. c. within 1 cm of each other. d. 2 cm difference.

c. within 1 cm of each other.

Anterior and posterior stability are provided to the knee joint by the: a) medial and lateral menisci b) patellar tendon and ligament c) medial collateral ligament and quadriceps muscle d) anterior and posterior cruciate ligament

d

Any lump found in the breast should be referred for further evaluation. A benign lesion will usually have 3 of the following characteristics. Which one is characteristic of a malignant lesion? a_ soft b) well-defined margins c) freely movable d) irregular shape

d

During an assessment of the spine, the patient would be asked to: a) adduct and extend b) supinate, evert, and retract c) extend, adduct, invert, and rotate d) flex, extend, abduct, and rotate

d

During the examination of the breasts of a pregnant woman, you would expect to fine: a) peau d'orange. b) nipple retraction. c) a unilateral, obvious venous pattern. d) a blue vascular pattern over both breasts.

d

The reservoirs for storing milk in the breast are: a) lobules b) alveoli c) Montgomery's glands d) lactiferous sinuses

d

The timing of joint pain may assist the examiner in determining the cause. The joint pain associated with rheumatic fever would: a) be worse in the morning b) be worse later in the day c) be worse in the morning but improve during the day d) occur 10 to 14 days after an untreated sore throat

d

During an assessment of the spin, the patient would be asked to: a. abduct and extend. b. supinate, evert, and retract. c. extend, adduct, invert, and rotate. d. flex, extend, abduct, and rotate.

d. flex, extend, abduct, and rotate.

The timing of joint pain may assist the examiner in determining the cause. The joint pain associated with rheumatic fever would: a. be worse in the morning b. be worse later in the day c. be worse in the moring but improve later in the day. d.occur 10-14 days after an untreated sore throat.

d.occur 10-14 days after an untreated sore throat

Social networks

informal supports accessed by older adults, such as family members and close friends, neighbors, church societies, neighborhood groups, and senior centers

Retraction

moving a body part backward and parallel to the ground

Protraction

moving a body part forward and parallel to the ground

Adduction

moving a limb toward the midline of the body

Functional ability

the ability of a person to perform activities necessary to live in modern society; may including driving, using the telephone, or performing personal tasks such as bathing and toileting

Social domain

the domain that focuses on an individual's relationships within family, social groups, and the community

Caregiver burden

the perceived strain by the person who cares for an older, chronically ill, or disabled person


Set pelajaran terkait

CCNA 1 (v5.1 + v6.0) Chapter 3 Exam Answers 2018

View Set

CPSC 317 Internet Computing: Module 4 (Socket programming with TCP)

View Set

ACE Personal Trainer Manual: Chapter 5

View Set

Econ 401 Exam 4- Marc Herold UNH 2019

View Set

Big Ideas Math Blue Book Chapter 5 - Systems of Linear Equations

View Set

A&P - Ch. 6 - Bones & Skeletal Tissue - Pt. 2 (2023)

View Set